You are on page 1of 43

lOMoARcPSD|10025139

Biomechanics Detailed Final Exam Notes

Introduction To Biomechanics (Western Sydney University)

StuDocu is not sponsored or endorsed by any college or university


Downloaded by masoud masoudi (masoudmasoudi9777@gmail.com)
lOMoARcPSD|10025139

Introduction to Biomechanics – Exam Study Questions


INTRODUCTION TO BIOMECHANICS:
1. Define Biomechanics.
 Application of mechanical principles concerned with internal & external forces/effects these
forces produce on humans. Biomechanists use mechanics (analysis of actions of forces) to
study anatomy/physiology of humans.

2. List 5 professional groups who benefit from a knowledge of Biomechanics


 Physiotherapists: assist in teaching recovery exercises, e.g. how to walk most effectively
 PDHPE Teachers: assist in teaching sport technique, e.g. volleyball serve
 Fitness Instructors: to maximise fitness benefits, e.g. best technique to do dead lifts
 Coaches: maximise training effect of drill/exercise, e.g. best way to pass a football
 Podiatrists: assists in clients not developing foot pain, e.g. proper walking techniques

3. Define and explain Quantitative Analysis in biomechanics


 Involving the use of numbers such as the height of a long jump. Used by researches,
coaches, performance analysts.

4. Define and explain Qualitative Analysis in biomechanics


 Involving non-numerical description of quality such as commenting on the technique of a
long jumper. Although it seems general, it is important to complement qualitative analysis with
quantitative analysis to more thoroughly see the ‘whole picture’ rather than just having a
number/measurement.

5. Explain the difference (using examples) between Kinematics and Kinetics in Biomechanics
 Kinematics: describes the appearance of motion including the pattern and speed of m/ment
sequencing by the body segments that allow understanding of degree of coordination.
 Kinetics: study of forces associated with motion such as analysing whether amount of force
generated by muscles is optimal for intended purpose of m/ment.

6. List the SI units for each of the following quantities: length, mass, time & temperature.
 Length: metre (m)
 Mass: kilogram (kg)
 Time: second (s)
 Temperature: degrees Kelvin (K)

7. Define Linear Motion and provide two examples.


 Linear Motion (translation) is in a straight (rectilinear) or consistently curved (curvilinear) line,
e.g. a skier coasting in a locked static position down a hill is in rectilinear motion BUT if
skier jumps over a gully with all body parts moving in same direction/speed along
curved path is curvilinear.
 Pure Linear Motion is all parts of the body moving in the same direction at the same speed,
e.g. a sleeping passenger on a smooth airplane flight is being translated through air.

8. How many types of Linear Motion are there?


 2: Rectilinear & Curvilinear

9. Define Angular Motion and provide two examples.


 Angular Motion (rotation) involves rotation around a central line/point (axis of rotation). All body
parts move through same angle/direction/time. E.g. gymnast performs a giant circle on the
bar. E.g. Springboard diver executes a somersault in midair.

10. Define General Motion and provide two examples.

Downloaded by masoud masoudi (masoudmasoudi9777@gmail.com)


lOMoARcPSD|10025139

 General motion is a complex combination of linear & angular motion. E.g. a football kicked
end over end translates through air as it rotates around a central air. E.g. A Frisbee
translates/spins.

11. Which type of motion (Linear, Angular or general) is most common in human movement?
 General motion categorises most human m/ment.

12. Describe the anatomical position.


 Standing erect, arms by side, palms facing forward,
feet slightly apart. Not a natural standing position.
Used as a reference position for defining m/ment
terms.

13. Why is a standard anatomical position used in


biomechanics?
 Anatomical reference point is used as the
position/starting place when m/ment terms are
defined.

14. Three anatomical reference planes divide the body.


Name, define and provide a movement example for
each of these planes.
 Sagittal (Anteroposterior/Median): divides body into left &
right. Forward/Backward m/ment, e.g. running
(arms/legs move forward/backward)/forward roll
(body moves parallel to Sagittal plane). Sagittal plane
m/ment: flexion, hyper/extension, dorsi/plantarflexion.
 Frontal (Coronal): divides body into front & back. Side-to-
side/lateral, e.g. cartwheel/jumping jacks. Frontal plane
m/ment: ab/adduction, lateral flexion, elevation,
depression, in/eversion, ulnar/radial deviation.
 Transverse (Horizontal): divides body into top & bottom.
Twisting motion, e.g. gymnast pirouette, ice skater
spinning. Transverse plane m/ment: left/right/medial/
lateral rotation, supination, pronation, horizontal
ab/adduction.
 Used to describe gross body m/ment, define more
specific m/ment terminology. These 3 cardinal planes intersect at a single point→ centre of
gravity. Other plane m/ment: Circumduction (general motion involving circular m/ment of body
segment).

15. Three anatomical reference axes bisect the body. Name, define and provide a movement
example for each of these planes.
 Mediolateral (Frontal-Horizontal): perpendicular to Sagittal plane, e.g. Knee extension
(kicking)
 Anteroposterior (Sagittal-Horizontal): rotation in frontal plane, e.g. jumping jack (major axes
of rotation passing through shoulders/hips)
 Longitudinal (Vertical): m/ment in transverse plane, e.g. throwing a Frisbee

16. Mass is a measure of what?


 Mass is quantity of matter in an object (kg).

17. Define Inertia.


 Resistance to action or to change. Tendency of a body to resist a change in its state of motion.
Directly proportional to mass→ ↑ mass ↑ chances of staying in current state of motion.

Downloaded by masoud masoudi (masoudmasoudi9777@gmail.com)


lOMoARcPSD|10025139

EXTRA LECTURE/TUTORIAL NOTES (Week 1):


 Statics: deals with systems in constant state of motion (either at rest [no motion] or moving with
constant velocity).
 Dynamics: deals with systems in which there is inconsistent motion (acceleration).
 Anthropometry: measurement of dimensions & weights of body segments (size, shape &
weight) in kinetic analysis.
 Kinesiology: study of human m/ment from perspective of science/art (biomechanics of human
m/ment is part of this.
 Mechanical systems determine nature of m/ment, e.g. throw: body (general motion), arm
(angular) & released ball (linear). Chosen by m/ment analyst depending on focus of problem
being studied.
 Spatial Reference System: Quantitatively describes m/ment; used to standardise
measurements taken. Cartesian Coordinate System: units are measured in the directions of
either 2/3 primary axes.
o 2D = x (horizontal) & y (vertical) coordinates. M/ment primarily in single direction
(planar), e.g. running, cycling, jumping. Points of interest are usually body’s joints
which constitute end points of body segments→ location of each joint centre can be
measured with respect to 2 axes (position of x is measured in contrast to y) – can be +/-.
o 3D= x, y & z (3D analysis). Z-axis is added perpendicular to x & y & measuring units
away from x, y plane in z direction. Z (vertical), with x & y (2 horizontal directions)
 Force: push/pull acting on body. Each force is characterised by magnitude, direction & point of
application. Body weight, friction, air or water resistance are all forces acting on body. F (force
in Newtons [N])=m (mass in kg) X a (acceleration in ms-2). Net force is sum of all acting forces.
 Weight: amount of gravitational force exerted on a body. Weight is equal to mass multiplied by
acceleration of gravity. Wt = mag
 Weight Force: force due to gravity. W = m x 9.8 (on earth).
 Pressure: force distributed over a given area; force per unit of area. P (pressure in Pa)= F
(Force in N)/A (area in m2).
 Volume: amount of space occupied by an object. L or mL.
 Density: mass per unit of volume. Density (p) = mass/volume.
 Specific Weight: weight per unit of volume (N/m2). Because weight is proportional to mass,
specific weight is proportional to density.
 Torque: rotary/turning effect of a force. Angular equivalent of linear force. T= F (force in N) x D
(distance in m).
 Impulse: product of force & time over which the force acts. I = FxT. A change in motion→ small
force acting for relatively long time or large force acting for relatively short time.
 Loads Applied to Humans:
 Compression: pressing/squeezing force
 Tension: pulling/stretching force
 Shear: force applied parallel to surface, e.g. suffering ACL snap
 Mechanical Stress: distribution of force within an object, quantified as force divided by area
over which force acts. Can be compressive, tensile/shear
 Bending: compressive stress on 1 side & tensile (tension) stress on the other
 Torsion: load-producing twisting around the objects longitudinal axis
 Combined Loading: simultaneous action of more than 1 loading force
 Acute Loading: single force applied once
 Repetitive/Chronic Loading: single force applied repeatedly
 Body Segment Interactions: A no. of body segments are moved in co-ordinated & sequential
manner to obtain maximum force/speed of m/ment at most distal end (e.g. throwing, kicking).
The contribution of each segment not only depends on muscle strength spanning each
proximal joint & the rotational radius, but timing of pattern of acceleration/deceleration. Forms
basis of Kinetic Link Principle: body segments generate high end point velocity by
accelerating/decelerating adjacent links in a sequential manner from proximal to distal, biggest

Downloaded by masoud masoudi (masoudmasoudi9777@gmail.com)


lOMoARcPSD|10025139

to smallest & from most fixed to most free, e.g. cracking a whip – transfer of momentum
(M=m x v) – end point breaks sound barrier [330m/s])

 Standard Notation: 123kg. Scientific Notation: 1.23 x 102 kg. Quantities are best written b/w 1-
10 multiplied by power of 10.
 Measurement Error: every person who operates an instrument is subject to experimental error
(in reading, setting or mistakes).
o Random Errors (noise): vary in magnitude/direction, e.g. air currents over scales
when measuring mass. Can occur due to lack of skill in the experimenter/sampling
procedure. Can be reduced by averaging results but not eliminated
o Systematic Errors: occur due to incorrect calibration, unjustified theoretical assumption
& incorrect use of apparatus. Difficult to detect. 2 types:
 Constant Errors: always same magnitude/direction, e.g. due to variation in
temp of air
 Drift Errors: same direction but progressively change in magnitude, e.g. if object
to have mass determined is at diff temp to scales them mass would slowly
change until 2 temps equalise.
 Significant Figures: All non-0 digits, all 0’s b/w 2 non-0 digits, all 0’s to the right of decimal point
& immediately following a non-0 digit are significant. All other 0’s are not significant. Give
answers to 3 significant figures (written to 2 decimal places).
 Rounding Off in Physics: A single 5 (no other non-0 digits to it’s right) is removed. When
rounding off 5’s, round to the nearest even digit, e.g. 7.25 & 7.15 become 7.2.

Downloaded by masoud masoudi (masoudmasoudi9777@gmail.com)


lOMoARcPSD|10025139

VECTOR ALGEBRA:
18. Explain the difference between Scalar and Vector quantities and provide examples for each.
 Vector quantities have both magnitude & direction; measured in displacement, velocity force,
momentum, torque & weight, e.g. 2km West
 Scalar quantities have only magnitude & no associated direction; measured in mass, volume,
time, speed & energy, e.g. s=d/t

19. Why are arrows useful for representing vector quantities?


 Because arrows have both magnitude & direction, they represent vectors; arrow head
represents direction & arrow length represents magnitude.

20. When two or more vectors of the same quantity are added together, what can be
calculated?
 Resultant vector can be calculated. Resultant=single vector (representing
numerous forces). E.g. travel 2km north, 3km south=1km south

21. Describe the process of adding vectors to find the resultant vector.
 ‘Tip to Tail’ Method. Adding Vectors: join the tail of the next vector to the tip of the
previous vector. Drawing the resultant: Resultant vector always point from tail of the 1st vector
to the head of the last.

22. Is the order of adding vectors together important when calculating the resultant?
 The order of adding vectors makes no difference to the resultant

23. What is Vector Resolution?


 Is determining the perpendicular components of a vector quantity relative to a
plane/structure. Usually horizontal/vertical.

24. Pythagoras’ theorem states what?


 An expression of the r/ship b/w the hypotenuse & the other 2 sides of a right
triangle. The sum of the squares of the lengths of the 2 sides of a right triangle
is equal to the square of the length of the hypotenuse. C2 = A2+B2

25. When using trigonometry: Sine = ?; Cos = ?; Tan = ?
 Sin = Opposite/Hypotenuse – SOH – VERTICAL VELOCITY
 Cos = Adjacent/Hypotenuse – CAH – HORIZONTAL VELOCITY
 Tan = Opposite/Adjacent - TOA

LINEAR KINEMATICS:
26. Motion that occurs in a straight line is called what?
 Linear Motion. Linear kinematics has no reference to forces; involves study of shape, form,
pattern & sequencing of linear m/ment through time.

27. Translational motion occurs when?


 Occurs in a straight line when all points on a body or an object moves the
same distance over the same time

28. Define linear DISTANCE, provide its symbol/s, unit of measurement and
state whether it is a scalar or vector quantity.
 Length b/w a start & finish point (along a pathway from A to B). Symbol: l (or
d). Unit: metre (m). Scalar quantity.

29. Define linear DISPLACEMENT, provide its symbol/s, unit of measurement and state whether it
is a scalar or vector quantity.

Downloaded by masoud masoudi (masoudmasoudi9777@gmail.com)


lOMoARcPSD|10025139

 Straight line distance b/w a start & finish point with its direction indicated (straight line from A to
B). Symbol: d (or s). Unit: m (+ direction). Vector Quantity.

30. Define linear SPEED, provide its symbol/s, unit of measurement and state whether it is a
scalar or vector quantity.
 Rate at which a body moves from 1 location to another. S=D/T. Symbol: s. Unit: m/s, ms-1, m.s-
1. Scalar Quantity.

31. Define linear VELOCITY, provide its symbol/s, unit of measurement and state whether it is a
scalar or vector quantity.
 Rate at which a body moves from 1 location to another in a given direction. Rate of change of
displacement. Velocity=displacement/time. Symbol: v. Unit: m/s, ms-1, m.s-1 (+ direction).
Vector Quantity.

32. What quantity does the slope of a displacement – time graph


provide?
 When plotting positions against the function of time on a time graph,
velocity can be calculated. Change in horizontal position (x axis) as a
function of time on a graph is called rise & change in time or vertical
position is called run. Slope measures: rise/run

33. The steeper the slope from a displacement – time graph would
indicate what?
 If slope is steep→ velocity is high b/w position changes (as it takes less time)
 If slope is flat→ velocity is low b/w position changes (as it takes more time)
 If there is no change in position (no rise) then slope & velocity is 0. Usually a + (right)/ – (left)
indicates direction of travel. During measurement values can fall b/w 1-4. Central difference
method gives velocity at a point in time (can be small intervals). Average velocity is calculated
using differences in position over 2 frames or larger time interval.

34. Explain the term instantaneous velocity.


 Velocity at a particular instant (small interval) in time, e.g. Javelin
release velocity. Average instantaneous velocity: Vav (Velocity
average) = (Vf [Final Velocity]-Vi [Initial Velocity])/2. Vav = (Vf-Vi)/2.
Secant: slope of line is calculated using central difference method
when calculating velocity over a time interval. Tangent: when
change in time becomes small (zero) the slope line touches the
curve once.

35. Define linear ACCELERATION, provide its symbol/s, unit


of measurement and state whether it is a scalar or vector
quantity.
 The rate of change in linear velocity. In human motion, the
velocity of a body/segment is rarely constant; changes are
due to forces that cause m/ment. Symbol: a. Unit: m/s².
Vector Quantity. Acceleration is the derivative of velocity.
Velocity is the 1st derivative of displacement.
Displacement is the area under a velocity curve. Area
under the curve is calculated by integration. Acceleration
is a 2nd derivative of displacement.
Average Acceleration: Vf-Vi / t2 –t1 OR ∆v (change in
velocity)/∆t (change in time). ∆v/∆t

36. If the velocity of an object is changing quickly, then the


acceleration of that object would be: High; Medium; or Low?

Downloaded by masoud masoudi (masoudmasoudi9777@gmail.com)


lOMoARcPSD|10025139

 Generally, acceleration means speeding up (↑ velocity), e.g. if motion in a straight line & in 1
direction (speeding up-acceleration/slowing down-deceleration). Thus, acceleration would
be high.

37. The slope of which type of graph indicates acceleration?


 The slope of a speed-versus-time graph indicates acceleration.

38. When interpreting the direction of acceleration, explain how we differentiate between an object
speeding up (accelerating) and slowing down (decelerating)?
 Problems arise when m/ment occurs in 1 direction then in the opposite direction. Thus, we
must assign directions to m/ment. Due to the direction, we label either + OR -, a + value of
acceleration may not indicate speeding up. Thus, + & - refers to direction of acceleration not
the type (speeding up/slowing down).
 Directional rules: Velocity is always in same direction as
displacement. If object/body is getting faster then acc is
in same direction as velocity. If object/body is slowing
down then acc is in opposite direction to velocity. Right =
+ direction. Left = - direction. Acceleration may be +, - or
0 based on direction of motion & direction of change in
velocity.

39. What might an acceleration = 0 mean for a particular


body?
 An acceleration of 0 does not mean the object has no
velocity. It indicates the object may not be speeding up
or slowing down. Thus, Acc=0 = constant velocity.

 Equations of Uniformly Accelerated Motion:


o Vf = Vi + at (at- acceleration x time)
o S (displacement) = Vit + 1/2at²
o Vf² = Vi² + 2as

PROJECTILE MOTION:
40. Any object thrown into the air is known as what?
 Any object thrown into the air is known as a Projectile.

41. The flight pathway of a projected object is referred to as


the what?
 The path taken by that projectile through the air is known
as the trajectory.

42. During human projectile motion, which body location is generally used for analysis?
 Centre of gravity is the body location used as reference for analysis.

43. In true projectile motion (objects in freefall), which two external forces act on the projectile?
 Air resistance & gravity. Thus, airplanes don’t count; influenced by forces generated from
engine.

44. During projectile motion, which flight component does gravity affect?
 Vertical component is influenced by acceleration due to gravity (constant downward force=
9.81m/s/s. Up is + & Down is -; thus a= -9.81m/s/s.) Gravity affects height travelled.

45. During projectile motion, which flight component does air resistance affect?
 Horizontal component is influenced by air resistance which affects distance travelled.

Downloaded by masoud masoudi (masoudmasoudi9777@gmail.com)


lOMoARcPSD|10025139

46. When ignoring air resistance, what affect does the size, shape and mass of an object have on
the acceleration of that object due to gravity?
 Acceleration due to gravity is independent (not dependant) of size, shape & mass. Acceleration
remains constant regardless of size, shape or weight of projectile.
 Vertical component of initial vel determines max height. As projectile gets higher its vertical vel
(Vv) ↓. At max height, Vv is 0. As object falls, Vv in the – direction in negative (-) direction is ↑.
Under normal conditions, air resistance can influence the vertical distance travelled.

47. The flight path of a projectile that is not influenced by air resistance would follow a trajectory of
which shape?
 Without air resistance, parabola shape trajectory. Parabola is curved with an apex;
symmetrical.

48. A projectile at the apex of its flight is represented by a vertical velocity of?
 Vertical velocity of 0; projectile completely stops moving for a period of time when reaching its
apex.

49. For a projectile that is released from and lands at the same height (ignoring air resistance), the
time taken for that projectile to reach the apex of its flight is equal to what percentage of the total
flight time?
 50%; without air resistance, there should be equal time from release point to apex &
consequent apex to landing point.

50. List three release factors that affect the trajectory of a projectile.
 Without gravity: object travel indefinitely with no change in vel (in space)
 With gravity: motion pathway changes vertically
 With air resistance: changes horizontal velocity (& vertical). Tail
wind ↑ horizontal displacement & ↓ release velocity. Head wind
↓ displacement & ↑ release velocity.
 Angle of Projection: relative angle of release
 Speed of Projection: relative speed of release
 Height of projection: relative height of release

51. Define Projection Angle.


 Angle at which a body is projected with respect to the
horizontal. Influences the shape of the projectiles trajectory. In
absence of air resistance, vertical, parabolic & horizontal (half
parabola) trajectory.

52. In the absence of air resistance, a projectile that is launched


from a height of 2.5 m with a projection angle of 0o will produce a
flight trajectory of which shape?
 Horizontal trajectory (half parabola)

53. Define Projection Speed and state which components of a


projectiles trajectory that it can influence.
 The magnitude of the projection velocity determines the length
or size of a projectile’s trajectory. Influences both height &
horizontal length of projectiles trajectory. E.g. when a body is
projected vertically upward, the projectile’s initial speed
determines height of apex. For a body that is projected at
an oblique angle, speed of projection determines both
height & horizontal length of trajectory.

Downloaded by masoud masoudi (masoudmasoudi9777@gmail.com)


lOMoARcPSD|10025139

54. What is meant by the relative height of release (Height of Projection)?


 Projection height is diff b/w height at which a projectile is released & height at which it lands. ↑

55. What effect does increasing the height of projection have on a projectile?
 Increasing Projection height→ ↑ time of flight→ greater range (horizontal distance).

56. How can maximum range / distance be achieved in projectile motion?


 Maximise speed of projection & release height without loss in release speed.

57. Describe how the optimum release angle for a projectile changes depending on whether the
release height is: greater than; equal to; or less than the landing height.
 When release height is 0 then optimal release angle is 45°. As release height ↑ (above landing)
then optimal release angle ↓. As release
height ↓ (below landing) then optimal
release angle ↑.
 Ball thrown up with a large horizontal
angle will reach more height but less
distance & vice versa. Object projected
upward in a vertical direction; speed will
determine height of apex. If projection
angle is oblique, effect of projection speed
will determine height & horizontal length
(range).
 E.g. Long Jump; in theory 45° is
optimal but to achieve this speed,
athlete would suffer; speed has been
shown to be a more important
determinant. Thus, actual angles of
elite (18-27). High Jump (40-48); maximise vertical displacement. Throwing (Shot Put
Elite:36-37); optimum angle of release must not restrict release speed.
 Positive angles of projection indicate angles of
greater than 0, i.e. object projected above
horizontal. However, tennis can be projected
downward from point to impact, e.g. slice shot (-3
to 15)

58. Under what condition can the equations for


uniformly accelerated motion be applied?
 Laws of uniformly accelerated motion used when a
body moves with a constant acceleration (+,- or 0);
laws of constant acceleration. Variables used
are displacement, velocity, acceleration &
time. Contains combos & provides flexibility
for solving problems where 2 quantities are
known & we have to solve for a 3rd.
 Analysing Projectile Motion: Initial vel has
both speed (magnitude) & angle of projection
(direction) = single quantity vector. Both vertical/horizontal components have certain speeds in
either direction. Assume that vertical/horizontal projectile velocity is constant & acceleration is
constant (0) throughout. Vertical/Horizontal is constantly changing due to
gravity (acc= =9.81 m/s/s).

ANGULAR KINEMATICS:
59. Define and list several important features of the Centre of Rotation.

Downloaded by masoud masoudi (masoudmasoudi9777@gmail.com)


lOMoARcPSD|10025139

 Difficult to locate moving axis of rotation but need to identify so angles can be measured. Joint
motion often accompanied by displacement of 1 bone with respect to articulating bone at a
joint. Position of centre of rotation continuously changes. Can find instant centre of rotation of
joint form x-rays.

60. What is meant by the ‘Instant Centre’ of a joint?


 The centre of rotation (CR) at a given joint angle or instant in time (location changes through
motion); instantaneous joint centre. E.g. Knee joint (medial & lateral femoral condyles are
symmetrical); tibia rotates long axis & axis through the knee from front to back as knee
flexes/extends.

61. What are the three conventional units used to measure angular motion?
 Degree (°), Revolution (rev) & Radian (rad); 1 complete rotation of circle =
arc of 360° (2π radians)

62. Which unit of angular motion is most appropriate for use in biomechanics?
 Radian: measure of an angle at centre of a circle described by an arc; to
length of radius of circle; dimensionless

63. Explain how to convert degrees to radians and radians to degrees.


 1 radian = 57.3°
 To convert degrees to radians (DIVIDE by 57.3); mostly use this
 To convert radians to degrees (TIMES by 57.3)

64. Define Relative Angle.


 Angle at a joint formed b/w longitudinal axes of adjacent body segments
(measuring angle of 1 body segment relative to other body segment articulating at
joint); doesn’t describe position of angles in space. Measure on same side of joint.
Used to quantify ROM (anatomical position); calculated using Law of
Cosines

65. Full extension of a joint is considered to represent what angle?


 Full extension is 0°.

66. Define Absolute Angle.


 Angle measured is b/w the body segment of interest & a fixed reference
line (angle of inclination of a body segment with respect to an absolute
reference line [usually horizontal/can be vertical]; describes orientation
of segment in space). Must be measured in same direction.
 Calculating absolute angles involve 2 primary conventions: co-ordinate
system at prox or distal endpoints of segment & angle measured in
counter-clockwise direction from right horizontal. Need to define tangent
of angle

67. List 4 tools that are commonly used for joint angle data collection in
biomechanics.
 Goniometer/Protractor
 Electrogoniometer: potentiometer to measure joint angle. M/ment
conducts electrical charge. Inexpensive immediate output signal
recording to PC; difficult to set up over fat of muscle
 Motion Analysis System: Video (2D&3D) & Infra-Red; software
application (digitising), e.g. Cricket – Muralitharan
 Accelerometers: measures relative segment angles/acceleration of a

Downloaded by masoud masoudi (masoudmasoudi9777@gmail.com)


lOMoARcPSD|10025139

segment relative to another; computed for force vectors. Output signals can be uniaxial/triaxial;
expensive – immediate output to PC

68. Explain the convention known as the Right Hand Rule and when it is applied.
 Cannot graphically represent angular motion using lines with arrows (impractical); essential to
determine direction of rotation. Used right hand rule; fingers of right hand curled in direction of
angular motion/rotation then extended thumb points in direction of angular vector. Magnitude
indicated by its length; cannot add by normal head-to-tail method.

69. Define angular DISTANCE, provide its symbol/s, unit of measurement and state whether it is a
scalar or vector quantity.
 The sum of all angular changes that have occurred; ϕ (phi); °/rad; scalar

70. Define angular DISPLACEMENT, provide its symbol/s, unit of measurement and state whether
it is a scalar or vector quantity.
 Angle b/w initial & final positions of rotating body; Ө (theta) ; °/rad; vector
 Angular displacement = Ө final - Ө initial

71. Explain the convention used to define the direction of angular


motion.
 Clockwise: NEGATIVE (-)
 Counter-Clockwise: POSITIVE (+)

72. Define angular SPEED, provide its symbol/s, unit of


measurement and state whether it is a scalar or vector quantity.
 Angular distance travelled per unit of time; Ϭ (sigma); °/s & rad/s;
scalar
 Angular speed = angular distance/time

73. Define angular VELOCITY, provide its symbol/s, unit of


measurement and state whether it is a scalar or vector quantity.
 Rate of change of angular position (angular displacement); ω
(omega); °/s & rad/s; vector. Presents slope of angular position on
time graph. Instant angular velocity presents slope of tangent to
angular position on time graph.
 Angular velocity = change in angular position/change in time
 Angular velocity is POSITIVE if Ө final is greater than Ө initial. If
no change in angle, slope is 0 & angular velocity is 0

74. Define angular ACCELERATION, provide its symbol/s, unit of measurement


and state whether it is a scalar or vector quantity.
 Rate of change in angular velocity occurring at a given time; α (alpha); °/s² &
rad/s²; vector
 Direction of angular acceleration vector must be determined; POSITIVE ang
acc may mean ↓ ang vel in a NEGATIVE direction OR ↑ ang vel in POSTIVE

direction
 Angular acceleration = angular displacement/time

75. How can the linear distance of a point on a segment be calculated when
that segment is undergoing rotation?
 The linear distance of segment motion at any point can be calculated if we
know the radian (r; measure of angle at centre of circle subtended by an arc
of length – radius of a circle = Ө = s/r; thus length of arc is s= r Ө) of the
point to the axis of rotation & the angle through which the segments rotates.

Downloaded by masoud masoudi (masoudmasoudi9777@gmail.com)


lOMoARcPSD|10025139

Thus, if length r1 rotates about elbow joint, the arc of rotation or distance the wrist moves is s 1
(distal point; travels greater distance than point closer to axis rotation) & the angle is Ө, then
linear distance is s1= r1 Ө
 Linear Displacement (curve) = radius of rotation x angular displacement
o R = radius of rotation; distance of the point from axis of rotation
o S = ‘length’ of curved path followed by the point (must have
same units as r)
o Ө in radians
 Angles has units of length (m) for linear distance but the radian is
dimensionless; rad x m = m

76. Define TANGENTIAL VELOCITY, provide its symbol/s, unit of


measurement and state whether it is a scalar or vector quantity.
 Tangential Velocity is product of distance of the point from the axis of rotation & the angular
velocity of the body. Linear velocity vector will represent the instantaneous tangent to the path
of motion referred to as the tangential velocity (VT). Linear velocity vector behaves as a
tangent; touches curved path at 1 point & is perpendicular to rotating segment.
 VT = radius of rotation (r) x angular velocity (ω); tall people have advantage (longer arm [lever]
however slower) & golf stick; longer stick ↑ power (distance) but lose accuracy is too long.
 E.g. Arm segment of length r=0.13m rotated with ang vel of 2.4 rad/s; the wrist vel will be
calculated as 0.13 x 2.4→ VT = 0.31 m/s

77. Explain how a hammer thrower could increase the tangential velocity of the hammer on
release.
 ↑ radial length (distance b/w centre of rotation)→ ↑ tangential/radial acceleration & ↑ angular
velocity

78. Define RADIAL ACCELERATION, provide its symbol/s, unit of measurement and state
whether it is a scalar or vector quantity.
 For an object following curved path, tangential linear velocity may not change in size but does
change in direction, e.g. baseball follows a curved path because of rotation of segments
before release (slightly inward & vertically downward); ball will accelerate downward &
inward toward shoulder or axis of rotation before release. The radial component (radial
acceleration) represents change in direction. ↑ in linear velocity or ↓ in radius of curvature ↑
radial acceleration. Thus, the smaller the radius of curvature (the tighter the curve is), the more
difficult it is for a cyclist to negotiate curve at high velocity.

79. Radial acceleration is also known as what and in which direction


does it act?
 Centripetal Acceleration; is always directed toward centre of
curvature/rotation.

80. Define TANGENTIAL ACCELERATION, provide its symbol/s, unit


of measurement and state whether it is a scalar or vector quantity.
 Acceleration of a body in angular motion due to rotation of a
segment may be resolved into 2 perpendicular components: at
tangent to angular acceleration (tangential component) & into
centre of rotation (radial component). Path of angular acceleration
follows a tangent & is usually perpendicular to rotating segment.
Tangential acceleration expressed in terms of radius of rotation &
ang acc (at = r α (angular acc). Important in discus throwing; time
rate change in the tangential velocity (Vt) of the discus along its
curved path is the tangential acceleration (at).
 Angular acceleration: change in angular velocity/time – α = ωf – ωi
/t

Downloaded by masoud masoudi (masoudmasoudi9777@gmail.com)


lOMoARcPSD|10025139

 Tangential acceleration: rate of change in tangential vel of a body following curved path; a t = Vtf
– Vti / t
 If at = r α (angular acc), then aR = r ω² (angular velocity) but can also be expressed as a
product of tangential velocity & radius of rotation; aR = Vt² / r. Tangential acceleration (at) &
Radial Acceleration (aR) are components of linear acceleration & are perpendicular to each
other→ resultant acceleration (aResultant = √ at² + aR²); units are angular velocity & acceleration
are rad/s or rad/s²

EXTRA LECTURE/TUTORIAL NOTES (Week 4):


 Measuring angles involves digitising; quantitative analysis of human rotational angular motion
involves filming athlete for each frame of film marking joint centres with dots & joining dots
together with straight line→ protractor (manual)/co-ordinate geometry (computer). Measuring
angular kinematics involves taking into account displacement, velocity & acceleration.
 Angular motion similar to linear motion, but ANGLES instead of linear position
 Low Extremity Joint Angles EXAMPLES:
o Ө hip = Ө thigh - Ө trunk. If hip angle is + then action is FLEX, if – action is EXT & at 0
in neutral→ Walking at moderate pace; hip angle ±20° & running; ±35°
o Ө knee = Ө thigh - Ө leg. Knee always in + or some degree of FLEX (- indicates
hyperextension; injury [outside of normative value range]) & at 0 in neutral→ walking;
knee 0-50° & running; 0-80°
 Relationship b/w Linear & Angular Motion: In human m/ment; results of m/ment are linear
whereas nature of m/ment resulting in motion of segments are angular, e.g. baseball pitcher
throws ball that travels linearly BUT motion of pitcher’s segment in the throw are
rotational.
 Both tangential & radial components of motion contribute to resultant linear velocity (V t) at
release, e.g. somersault dismount; primary contribution to linear velocity of body’s COG
is from tangential acceleration BUT 50% of velocity can be from radial component.
 The instant an object is released, tangential component of angular acceleration is 0 because
the thrower is no longer applying a force (as is the radial component); thus timing is critical.

LINEAR KINETICS:
81. Kinetics deals with which component of biomechanics?
 Causes of motions; from forces acting on a system

82. Define FORCE, provide its symbol/s, unit of measurement and state whether it is a scalar or
vector quantity.
 Any interaction, push/pull b/w 2 objects causing change in state of motion; could produce/stop
motion, + or – acceleration or change direction; F/f; Newtons (N); vector (have magnitude &
direction; use arrows – length indicates magnitude of force & arrow heads points in direction of
applied force)

83. Objects move when acted upon by a force greater than……..?


 Objects move when acted upon by a force greater than their natural resistance (Inertia;
momentum)

84. Explain what is meant by the ‘Point of Application’ of a force and provide an example.
 Specific point of applied force to an object, e.g. muscular
force is applied at centre of muscles (mm’s) attachment to
bone.

85. Explain what is meant by the ‘Line of Application’ of a force.


 Straight line of infinite length in direction of acting force; force
may produce same acceleration of object anywhere along line.

Downloaded by masoud masoudi (masoudmasoudi9777@gmail.com)


lOMoARcPSD|10025139

86. Explain what is meant by the ‘Angle of Application’ of a force.


 Orientation of line of application gives angular position; with respect to X-Y co-ordinate system

87. In relation to force systems, explain each of the following terms: Coplanar forces; Concurrent
forces; Collinear forces; force Resolution.
 Forces are vectors & have angle of application; may have no. of forces (resultant). Forces may
be resolved into components (horizontal & vertical) or into net effect of all forces in a system.
Force Systems: different/multiple forces created
 Coplanar: acting in a single plane; resultant force when adding vectors (head-to-tail method)
 Concurrent: acting at a single point
 Collinear: vectors act along a single line; add/subtract 3 or 4 forces depending on direction =
NET effect
 Resolution: break force into its components with respect to co-ordinate system. Vertical &
horizontal directions relative to axis of bone; use sin & cosine.

88. Newton’s 1st Law of Motion states that ……


 A body will maintain a state of rest/constant velocity (motion) unless acted on by an external
force that changes the state of motion

89. Newton’s 1st law of motion is also known as the law of what?
 The Law of Inertia. Inertia; reluctance to change state of rest/motion. Related to amount of
mass (measure of amount of matter in object); greater the mass, the greater the inertia (harder
to move). E.g. to lift weight, NET force must be greater than force of inertia. Acc can be –
(stop/slow down) or + (move)

90. Newton’s 2nd Law of Motion states that……


 A (NET) force applied to a body causes an acceleration of that body of a magnitude
proportional to the force in the direction of the force & inversely proportional to body’s mass. F
= mass x acceleration (F=ma). Newtown = kg/m/s². If forces counteract each other, net force =
0 similar to 1st law. If acceleration produced, object will travel in a line comparable to line of
action of net force; a = v/t. So if F=ma then f= m v / t. The product of mass & velocity is
momentum (quantity of motion of an object; momentum [p] = mass x velocity [p=m v]). To
change momentum, external force must be applied which will ↑ or ↓ momentum.

91. Newton’s 2nd law of motion is also known as the law of what?
 The Law of Acceleration.

92. Newton’s 3rd Law of Motion states that ……


 For every action (force) there is an equal & opposite reaction (force). When 1 body exerts a
force on a second, the second body exerts a reaction force that is equal in magnitude but
opposite in direction on the first body. Force never acts in isolation/always in pairs/nerve cancel
each other out.

93. Newton’s 3rd law of motion is also known as the law of what?
 The Law of Reaction

94. When considering Newton’s Laws, explain why a person who applies a force to the ground in a
vertical jump is projected upwards by the reaction force from the earth but the earth does not
appear to experience movement as a result of the initial downward force from the person?
 Effect of force greater on person because earth has greater mass.

95. Explain what is meant by ‘Non-Contact Force’ and provide an example.


 Non-contact forces acts at distance; forces that are exerted by objects not in direct contact with
1 another→ gravity

Downloaded by masoud masoudi (masoudmasoudi9777@gmail.com)


lOMoARcPSD|10025139

96. The Law of Gravitation states that ……


 All bodies are attracted to 1 another with a force proportional to the product of their masses &
inversely proportional to distance b/w them. Fg (force of gravitational attraction)= G (universal
gravitational constant; 6.67 x 10-11 Nm²/kg²) m1 (mass of 1 object) m2 (mass of the other object)
/ d² (distance b/w the mass centres) = Fg=Gm1m2 / d². The greater the mass of either body, the
greater the attractive force between the two; similarly, the greater the distance b/w bodies, the
smaller the attractive force b/w them. Only gravitational attractions involving earth are
considered in biomechanics→ rate of gravitational acceleration at which bodies are attracted
towards surface of earth (9.81 m/s²) is based on earth’s mass & distance to centre of earth.

97. How is the Weight Force of an object calculated and what units is it measured in?
 Attractive force of earth on an object (weight of an object); W = ma – where a is 9.8 m/s² or W
= mg – where g is acceleration due to gravity. Body weight is product of individual’s mass &
acceleration due to gravity (mass & body weight not the same). Weight is a force & vector; has
line of action (COF to centre of earth, vertically downward) & a point of application (COM of
person).

98. Explain how altitude can affect acceleration due to gravity and subsequently the weight force
of and object.
 Gravitational force is less when the distance b/w earth & an object are greater. Thus higher
altitude (acceleration due to gravity decreases) = less gravitational force. Less gravitational
force due to higher altitude (close to equator; centre of earth mass), since greater altitude
means greater distance from the Earth’s centre. Altitude can affect value of a or g; higher the
altitude, the lower the value for g – 9.78 m/s² at equator & 9.83m/s² at poles.

99. What biomechanical implications does higher altitude have for sporting performance?
 Higher altitude means less gravitational force, thus athletes can jump higher & throw further.

100. Explain what is meant by ‘Contact Forces’ and provide an example.


 Involves actions, pushes or pulls; exerted by 1 object in direct contact with another object, e.g.
bat & ball. These contact forces are paramount: Ground Reaction Force, Joint Reaction Force,
Friction, Fluid Resistance, inertial force, muscle (mm) force & elastic force.
 Joint Reaction Force: net force acting across a joint; Newton’s 3rd law, e.g. while standing;
thigh exerts downward force on leg across knee joint whilst the leg exerts an upward
force on thigh of equal magnitude.

101. GRF is the abbreviation used to describe what property?


 Ground Reaction Force: Newton’s 3rd law; Vertical Component (stress on bones/mm) &
Horizontal Component (horizontal motion).

102. List several factors that can affect GRF’s during running.
 Running style (rearfoot, midfoot & forefoot), running speed, footwear, ground surface & surface
grade (uphill, downhill, inclination). Runners generally ↑ stride length to ↑ speed when going
from slow to moderate then ↑ stride rate; but longer strides tends to generate GRF’s with larger
retarding horizontal components; thus overstriding is counterproductive.

103. VGRF is the abbreviation used to describe what property?


 Vertical Ground Reaction Force; walking (1-1.2 BW) & running (3-5 BW) & graphically have
different shapes. Important in high jump, whereas horizontal
component in long/triple jump.

104. Indicate what each of the following represent: Fz; Fy; and Fx.
 Fz: vertical; up-down
 Fy: antero-posterior; forward-backward

Downloaded by masoud masoudi (masoudmasoudi9777@gmail.com)


lOMoARcPSD|10025139

 Fx: medio-lateral; side-to-side

105. GRF is usually measured in Newtons but can also be represented by what convention?
 Scaled by dividing by person’s body weight; in units of
‘times body weight’ (BW).

106. Define Friction.


 A force acting parallel to interface of 2 surfaces in
contact.

107. What affect does friction always have on motion?


 Friction always opposes motion; opposite in direction to
m/ment or tendency for m/ment. Magnitude of friction
determines relative ease/difficulty of moving the
contacting objects relative to each other, e.g. enough
horizontal force needed to move table, if too small,
will not move

108. What is meant by Maximum Static Friction Fm?


 Static Friction (Fs): small horizontal force, equal but
opposite friction, weight & reaction force cancel→ no net
force→ no m/ment
 Max Static Friction (Fm): max amount of friction that can
be generated b/w 2 surfaces in contact. If applied
horizontal force is greater than Fm there will be m/ment.

109. The friction present once one object begins moving


(sliding) over another is known as what?
 Once m/ment starts, the friction present is kinetic friction
(Fk); will have constant magnitude only when there is m/ment (will stop when m/ment stops).
Regardless of amount of force applied; Fk force remains the same.

110. For two surfaces, will the maximum static friction between them always be greater than the
maximum kinetic friction?
 Fm will always be greater than that of Fk as once Fm is larger than Fk, movement begins & the
force remains greater whilst m/ment occurs. If Fm is not great enough, the object would not
move and there would not even be Fk. Fk < Fm

111. What two factors determine the amount of friction between two surfaces?
 Coefficient of Friction & Normal Reaction Force

112. Define the ‘Coefficient of Friction’ and indicate what it is influenced by.
 Coefficient of Friction (µ): unit less number that indicates relative ease of sliding OR amount of
mechanical/molecular interaction b/w 2 surfaces in contact. Influenced by roughness &
hardness of surfaces; influences type of molecular interaction b/w surfaces – depends on both
surfaces. ↑ roughness to ↑ friction (↑µ) & ↑ smoothness to ↓ friction (↓µ).

113. In relation to friction, what is meant by the term ‘Normal Reaction Force’?
 Normal Reaction Force (N or R): if weight is only vertical force then N is equal in magnitude to
weight & opposite in direction. If there are additional vertical forces, then magnitude of N is
vector sum of all vertical forces (or vertical components of forces acting at an angle). Thus, as
µ remains constant, Fk & Fm are lower when the object is pulled because pulling is easier than
pushing. ↑ force to ↑ friction (e.g. add weight in abseiling) & ↓ force to ↓ friction (e.g. subtract
weight in formula 1 driving)

Downloaded by masoud masoudi (masoudmasoudi9777@gmail.com)


lOMoARcPSD|10025139

114. Does the surface area in contact between two surfaces affect the amount of friction between
them?
 Surface area DOES NOT alter total friction. ↑ contact area associated with ↑ weight thus ↑
friction due to ↑N. (e.g. ↑ tyre area means wear is reduced)

115. How does rolling friction compare with sliding friction?


 Rolling friction b/w rolling object & a flat surface. Influenced by
weight, radius & deformability of rolling object, as well as by
coefficient of friction b/w 2 surfaces. 1/100-1000 less than sliding
friction.

116. What effect does lubricants have on the friction between two
surfaces?
 Lubricants/fluid b/w surfaces dramatically ↓ µ

117. Excluding the use of lubricants, explain how the friction between
two surfaces can be altered (Provide examples to support your answer)?
 Roughen surface to ↑µ; rough gloves in golf/weight training ↑ friction
thus ↑ grip & ↑ performance
 Smoothen surface to ↓µ; smooth dancing shoes allowing
easier gliding/ment by ↓ friction

118. What is a Free Body Diagram and when are they used?
 The 1st step in analysing forces on body/system; there will
be a no. of forces acting simultaneously (internal/external).

 Mechanical Analysis: Allows understanding of cause-effect


r/ship & effects of force at instant time (sum of forces), applied
over time (impulse) & over a distance (work).
 Static Analysis: systems at rest or moving at constant velocity;
all external forces (combined) acting on system are cancelled
(balanced); resultant = 0, e.g. rope pull
 Dynamic Analysis: accelerations are non-zero, e.g. catching a
ball – different forces of ball & forces applied to stop ball

LINEAR KINETICS:
119. Define linear Momentum, provide its symbol/s, unit of measurement and state whether it is a
scalar or vector quantity.
 Quantity of motion an object possesses; measured as the product of a body’s mass & its
velocity; affects outcome of interaction b/w 2 objects involving collisions; p or M; kg/m/s; vector.
 M = mass (kg) x velocity (m/s)

120. How can the momentum of an object be increased?


 ↑ velocity &/or ↑ mass. Change in momentum can be due to mass of object or velocity. Head
on collision; m/ment in direction of object with greatest momentum. Other factors: becoming
entangled, falling over & elasticity; will produce a diff velocity.

121. Can an object that is stationary have momentum?


 No; static object has no momentum. If velocity = 0 then M = 0

122. Explain what is meant by ’Conservation of Momentum’.


 Newton’s 1st Law Modified – Principle of Conservation of Momentum: in the absence of
external forces, the total momentum of a given system remains constant; sum of the
momentums before collision is sum of the momentums after collision. Pi = Pf OR Mvi = Mvf

Downloaded by masoud masoudi (masoudmasoudi9777@gmail.com)


lOMoARcPSD|10025139

123. Define linear Impulse; provide its unit of measurement.


 The effect of a force applied over a period of time; measure of what is required to change
motion (velocity) of an object (over a period of time which will change momentum of object).
Impulse = F x t. Measured in Newtons (N).

124. Impulse is represented by the area under which type of


graph?
 Impulse is area under a force-time graph/curve

125. Explain the Impulse – Momentum relationship.


 When external forces act they change the momentum in a system. Impulse = momentum
change. Depends on magnitude of forces & time over which they act. Ft = mvf – mvi. E.g.
person with mass of 65kg jumps from a squat position into air. Initial vel was 0 & Final
Vel was 3.4 m/s; Ft = 65x3.4 – 65x0 = 221 kg/m/s. If force application took place over 0.2
seconds, the average force applied would be F x 0.2s = 221 (REARRANGE); F=2212/0.2 =
F=1105 N
 Thus, if joints of lower extremity are flexed on landing, time over which contact forces act will
be extended. If land over same time – greater impulse in (stiff leg) condition as less time to
dissipate F.

126. Explain how the Impulse – Momentum relationship can be manipulated to gain a performance
advantage.
 ↑ force (F) & ↓ time (T) = ↑ momentum. Vertical jump: during take off; if T is ↑ then amount of F
generated ↓. Need to maximise impulse through optimising F & T (correct timing). Effective
time of flexion at knee, ankle & hip will produce effective force. However, high force over short
time will result in higher injury risk.

127. An Impact is defined as a collision between two bodies that is characterised by what?
 The collision of 2 bodies characterised by large forces exerted by 2 bodies over a small time
interval. Behaviour of objects after collision depends on collective momentum & nature of
impact.

128. Most impacts lie on a continuum between two theoretical endpoints


that are represented by what type of collisions?
 Perfectly Elastic Impact: relative velocities before = relative velocities
after. Impact during which velocity of system is conserved. If rebound,
then e=1, more elastic impact, energy retained, high velocity. High e =
warm squash ball
 Perfectly Plastic Impact: at least 1 object deforms & doesn’t return to
original shape & objects don’t separate. If no rebound, then e=0, more plastic impact, energy
not retained, low velocity. Low e = cold squash ball
 Most impacts are somewhere b/w the 2 indicated by coefficient of restitution.

129. What does the ‘Coefficient of Restitution (e)’ represent?


 If 2 bodies move towards each other along the same straight line, the difference b/w their
velocities immediately after impact bears a constant r/ship to the difference their velocities at
the moment of impact; it is the interaction b/w 2 bodies during impact (it doesn’t describe the
object/surface involved).
 In any impact situation, the collision outcome depends upon manner in which the colliding
objects are moving before impact & on their respective
elastic properties (measured by e). E describes the elasticity
of an impact & is relevant in situations where we are
interested in the velocity separation of 2 bodies (e.g. bat &
ball) after impact.
 Describes an interaction, not a property.

Downloaded by masoud masoudi (masoudmasoudi9777@gmail.com)


lOMoARcPSD|10025139

 Ball hits a fixed surface; both ball & surface are compressed at impact; 2 objects try & return to
their former shape. Elasticity: the relative velocities of the 2 bodies before & after impact are
the same. Inelastic collisions: one body in contact deforms (doesn’t regain original shape).
Colliding objects should generally be matched in the following way when striving for maximum
velocity of separation (hard against hard & soft against soft).

130. List and explain FOUR factors that can affect the coefficient of restitution between two
bodies.
 Properties/nature of materials that makes up object (how elastic)
 Temperature, e.g. squash ball that has higher temp will ↑ e & thus ↑ rebound from
racquet/wall
 Nature of collision: Velocity, Angle & Orientation of impact
 Loss of energy due to heat, sound & deformation
 Size/shape of objects

WORK, POWER & ENERGY:


131. Define mechanical Work; provide its symbol/s and unit of
measurement.
 Effect of force applied over a distance. W = Force (f) x
displacement (d); W; Joule (J)/Newton Metre (Nm). No force = No
m/ment = no mechanical work & distance moved is 0, e.g. force
needed to move object was 100 N moved 1m = 100Nm
 Work: force applied against a resistance multiplied by the displacement of resistance in the
direction of force; the quantity of energy transferred/transformed.
 Generally, W = F x coz (angle) x s (displacement); work done depends on direction of force.
E.g. If box pushed 0.1m with force of 50N parallel to surface – F x cos 0 (cos 0 = 1) x s→
50 x 1 x 0.1 = 5Nm BUT with same force but at angle of 30 & same distance – 50 x cos30
x 0.1 = 4.33 Nm

132. Is mechanical work done when a muscle contracts isometrically?


 No, isotonic muscle contraction is required→ mm tension→ m/ment→ mechanical work.
Isometric contraction→ no work→ no m/ment

133. Differentiate between +ve and –ve work.


 POSITIVE: force & m/ment (acceleration) in same direction; greater kJ (caloric expenditure)
than same amount of neg mechanical work (but no formula b/w energy used & biomechanical
work due to agonist & other muscle groups co-contracting & being difficult to monitor).
 NEGATIVE: net force & m/ment in opposite direction, applied & force to decelerate

134. Define Power; provide its symbol/s and unit of measurement.


 The rate at which a force does work; amount of mechanical work performed in a given time; P;
watts (W). As W = f x s & v = s/t then P = F x s / t (speed over time) as P = f x velocity
 P = Work (W) / t (time period work was done)

135. If work remains constant, what must be done to exert more power?
 If work remains constant, greater power needed to do mechanical work quicker. The amount of
work done & the time taken to do that work determines the power exerted. ↓ time to lift/move
object, ↑ power developed to perform work

136. Power sports such as jumping, throwing and sprinting require the ability to do what?
 Ability to exert a large force rapidly is crucial in performance. A combo of force & velocity
required for effective power.

137. Differentiate between Energy and Mechanical Energy.

Downloaded by masoud masoudi (masoudmasoudi9777@gmail.com)


lOMoARcPSD|10025139

 Energy: the capacity to do work


 Mechanical Energy: capacity to do mechanical work; 2 forms
 Types: light, heat, electrical & mechanical

138. What are the units used to measure energy?


 J (Joule)

139. Define Kinetic Energy; provide its symbol/s and unit of measurement.
 Energy resulting from motion; KE/Ek; J; scalar (not directional dependant = linear). A moving
body must have energy; a force must be applied to move/stop over distance.
 Ek = ½ mv². If v = 0 then Ek = 0. e.g. if a 2kg ball rolls with a velocity of 1m/s. Ek = ½ x 2kg
x (1m/s)² - 0.5 x 2 x 1² = 1J. Change in velocity ↑ amount of energy in object.

140. A moving body has kinetic energy and thus the capacity to do what?
 A moving body with Ek has the ability to do work from motion

141. Define Potential Energy; provide its symbol/s and unit of measurement.
 The energy possessed by a body due to it’s position above a reference surface (usually earth’s
surface/floor); object may have stored energy because of its height/deformation; energy of
position; PE/Ep; J.
 Ep = m x ag x h – mass x acceleration due to gravity x height above reference point
 Ep = weight (mass x gravity) x h

142. Potential is considered what kind of energy?


 Stored energy

143. Define Strain Energy; provide its symbol/s and unit of measurement.
 The capacity to do work because of a deformed body returning to its original shape; type of
potential energy; associated with elastic forces; if object deformed, resistance to deformation ↑
as object is stretched & released as elastic energy; SE; J.
 SE = ½ x k x x² - ½ x k (spring constant for the material; this quantifies the stiffness of material
& ability to store energy on deformation) x x² (amount of deformation; the distance over which
object is deformed). Depends on force & time periods as to how much SE can be generated

144. List 5 examples where strain energy plays an important role in movement performance.
 Stationary Equipment: diving boards, trampolines & running track (when surface is depressed,
SE created; subsequent conversion of stored energy to KE enables surface to return to original
shape
 Equipment in Motion: archery bows, sports balls & pole vaults (store strain energy as they
bend & then release KE & ↑PE of athlete as they straighten during performance of vault).
 Footwear: sports shoes; running
 MM on stretch prior to force generation; dynamic muscular contraction of
muscles initiate the stretch-shorten cycle whereby he eccentric phase
enhances performance in the concentric phase through elastic energy
storage.
 Maximum effort throw – stored energy in stretched musculotendinous
units contribute to force/power generated & resultant velocity

145. The Law of Conservation of Energy states that…


 When gravity is the only external force, a body’s mechanical energy
remains constant; there is a correlation b/w KE & PE of an object/body

146. Energy cannot be created or destroyed. Therefore how can it be gained or removed by a
body?

Downloaded by masoud masoudi (masoudmasoudi9777@gmail.com)


lOMoARcPSD|10025139

 It can be transformed from place to place; transformed from 1 type to another; KE + PE =


constant. Occurs in projectiles such as human m/ment or throwing ball up where only
force is gravity.
 ↑ height = ↑ PE but KE↓ due to ↓ acceleration/gravity. Apex – height & PE at max value but vel
& KE = 0
 Total energy of system never changes because of ↑PE as there is equal ↓ KE & vice-versa.

147. A special relationship exists between work and energy. What is this relationship?
 The work of a force is equal to the change in energy that it produces in the object acted on.
 W = KE + PE + TE (thermal energy or heat) – work = change in energy. The change in sum of
forms of energy produced by a force is quantitatively equal to the mechanical work done by
that force
 Sport: foot arches store SE as they flatten; this is released at toe off & contributes to desired
segment/whole body motion – shoe changes the SE in foot & can ↓ performance & damage
body; storage of E in arches & Achilles tendon reduces the amount of energy burned in
running.
 Sport: Mm that cross 2 joints contribute to mechanical energy. E.g. hip extensors
(concentric)/rectus femoris (isometric)/gastrocnemius – vertical jump take off related to
mechanical energy transfer; hip extension occurs, rectus femoris doesn’t change→
extensor torque at knee & gastrocnemius in conjunction with rectus femoris extend
distal joints by transferring mechanical energy from proximal to distal end; also
important in shock absorption (dissipating force from distal to proximal in injury prevention).
 Concentric: shorten due to force applied – energy developed – KE & PE gain
 Isometric: no change in length with force – no work – body remains the same
 Eccentric: lengthening with force applied – energy absorbed – loses KE & PE

TORQUE:
148. What is meant by the term ‘static equilibrium’?
 In static equilibrium, all external forces are balanced & the body is motionless, i.e. acc = 0 - ∑F
=0
 Similarly, in angular situations, if all external forces are balanced then there is no rotation, i.e.
angular acc = 0 - ∑T = 0

149. Define Torque; provide its symbol/s and unit of measurement.


 The rotary (turning) effect of a force about an axis of rotation; moment of force. When the line
of a force acts away from the centre of mass of an object; a torque/moment is produced that
causes rotation. Outcome: to produce rotation/pivoting about an axis/fixed point
 Torque = Force x d┴ (the perpendicular distance from the line of action of force to axis of
rotation) or Torque = F x Moment arm

150. The perpendicular distance from the line of action of a force


to the axis of rotation is known as the what?
 Moment Arm

151. Under what circumstances are torques produced?


 A force is applied causing rotation about a specific axis; torque is not a force,
rather torque is the effectiveness of a force causing rotation.

152. You have a flat tyre and are having great difficulty loosening a nut on your car
tyre. Explain how you could generate greater torque so that the nut can be
loosened.
 Extend/lengthen moment arm (longer spanner; greater distance b/w force applied & nut acting
as axis of rotation) will ↑ torque without necessarily ↑ force.

Downloaded by masoud masoudi (masoudmasoudi9777@gmail.com)


lOMoARcPSD|10025139

153. Explain why it is easier to push open a door by applying the force further from the hinge side
of the door.
 It is easier to initiate rotation when force is perpendicular to lever & as far away as possible
from axis of rotation. Moment arm = shortest perpendicular distance b/w force’s line of action &
it’s axis rotation; ↑ moment arm = ↑ torque

154. Explain the convention used to describe the direction of rotation caused by applying a torque.
 Clockwise = NEGATIVE
 Counter-Clock Wise = POSITIVE

155. Explain how / why the resistance changes during a bicep curl and at which point the
resistance is greatest.
 Torques in the Human Body: Moment Arm = perpendicular distance b/w joint centre & muscle’s
line of action. As joint moves through ROM there are changes in the MA’s length. During a
bicep curl, 90° is the point at which resistance is greatest as at this point, the MA is the greater
distance from the joint centre [humerus]; greater force is needed. At the elbow, as the angle
pull moves away from 90° in either direction, the MA for the elbow flexors is progressively
diminished. Since torque is the product of MA and muscle force, changes in MA directly affect
the joint torque that a muscle generates. For a muscle to generate a constant torque through
ROM, it must produce ↑ forces as MA ↓.
 Torques on Joints: The product of muscle tension & muscle
moment arm produces a torque at the joint crossed by the muscle.
Human m/ment involves simultaneous tension development in
agonist (causing motion) & antagonist (opposing m/ment; control
velocity of m/ment; enhance stability & create torques opposite to
agonist) muscle groups. As measuring muscle forces are
impractical, estimates of resultant joint torques using body segment
weight, motion & action of external forces to investigate m/ment
patterns.
 Concentric Motion: Net torque & joint m/ment in same direction
(Downward Phase of Bicep Curl; resistance is less as motion is
with resistance torque motion)
 Eccentric Motion: Net torque & direction of joint m/ment opposite
(Upward Phase of Bicep Curl; resistance is greater as force is
being generated against resistance torque).
 It assumed that muscular force (& subsequent joint torque)
requirements of resistance exercise ↑ as amount of resistance ↑;
this is only true if movement kinematics remain the same, as for e.g. use
of a wide stance compared to a narrow stance produces greater
torques at both hip/knee. When all else is constant; speed of m/ment ↑
during ↑ joint torques. But, ↑ m/ment speed during weight training is
undesirable as ↑ speed ↑ likelihood of poor technique & injury. As
muscle’s don’t need to work as hard for whole motion (↓ training
stimulus); slow controlled m/ment technique is recommended in weight
training so muscle is loaded (working) throughout whole ROM & not just ↑
workload for overcoming longest MA; ineffective muscle gain.
 Cycling: lower extremity joint torques affected by pedalling rate, seat
height, length of pedal crank arm, distance of pedal spindle to ankle joint.
Joint torque requirements ↑ as resistance ↑
 Rowing: Traditionally adjacent crew member’s row on identical opposite
sides of boat; but this created unequal moment arms (unequal forces; lateral oscillation);
variations in positioning eliminate this problem by producing equal forces on each side.

LEVERS:
156. What is a lever?

Downloaded by masoud masoudi (masoudmasoudi9777@gmail.com)


lOMoARcPSD|10025139

 A rigid bar; free to move about a point of suspension called a fulcrum. Force applied at 1 point,
called the effort is used to overcome another force at another point, called the load/resistance.
As with pulley system, large mechanical advantage can be obtained. Human m/ment mimics a
lever system.

157. Name the 3 main features found in a lever system.


 All bones are levers
 Joint in fulcrum
 Muscle/ligament is effort force
 Body parts (hand/foot/external object) to be moved as load

158. Name and describe (with the aid of diagrams) the 3 main
types / classes of levers systems.
 1st Order/Class Lever (See-Saw Type): fulcrum is b/w the load/resistance & effort as in a see
saw. Very few in human body, e.g. the skull-m/ment of head; load (head), fulcrum
(atlas/axial of vertebral column) & effort (sternocleidomastoid & other neck muscles)
 2nd Order/Class Lever (Wheelbarrow Type): load is b/w fulcrum & effort. Very few in body, e.g.
standing on toe’s; fulcrum (ball of foot), load (body weight acting through ankle bones)
& effort (gastrocnemius muscle)
 3rd Order/Class Lever (Fishing-Rod Type): effort b/w fulcrum & load as in
fishing-rod/spade. E.g. human forearm (holding); load (lower arm),
fulcrum (elbow) & effort (biceps muscle).

159. Which is the most common lever type in the human body?
 3rd Class Lever prominent in all joints of extremities.
 Torsional 4th Class Lever: load directly above fulcrum; force is 1 that
causes rotation, e.g. weight of skull acting on axis of rotation

160. What two mechanical advantages can lever systems provide?


 The ratio of effort arm to resistance arm – MA = effort arm/resistance
arm. Depends on relative distances b/w fulcrum & load; fulcrum & effort.
If mechanical advantage = 1 there is no advantage as effort arm equals
resistance arm; whereby the function of lever is to alter direction of
motion or balance lever
 Force advantage: If fulcrum to load distance < fulcrum to effort distance.
The load is greater than the effort applied as the load moves through
less distance than the effort in same time; force gain is paid for loss in
distance/speed.
 Speed/Distance advantage: If fulcrum to load distance > fulcrum to effort distance. The load
moves further than the effort point in the same time; distance/speed gain is paid for by a force
loss
 1st lever system can have both mechanical advantages or no mechanical advantage (3) but not
at the same time; e.g. musculoskeletal system can have MA of 1, < or > than 1 due to
action of agonist/antagonist muscles

161. What effect is produced by a lever system where the resistance arm is greater than the force
arm?
 ↑ speed/distance & ↓ force – greater force/effort needed to overcome resistant force;
velocity/speed of m/ment will ↑. Effort acts over small distance but resistance force is moved a
greater distance in same amount of time (<1).

162. What effect is produced by a lever system where the resistance arm is shorter than the force
arm?
 ↓speed/distance & ↑ force – torque created by force/effort is ↑ by greater effort arm (>1).

Downloaded by masoud masoudi (masoudmasoudi9777@gmail.com)


lOMoARcPSD|10025139

163. What mechanical advantage does a second class lever always


have?
 Force advantage; load is always closer to fulcrum than effort –
FL < FE – MA >1, e.g. calf raise

164. What mechanical advantage does a third class lever always


have?
 Speed/Distance advantage; load is further away from fulcrum –
FL > FE – MA <1, e.g. all joints of extremities require large
effort force to gain speed. Skilled athletes use anatomical
levers to improve performance; e.g. fully extend arm in tennis
serve to ↑ racquet speed which promotes improved
ROM/angular speed development of body segments→ when
muscle force greater than resistance force – POSTIVE
mechanical work

 The angle at which a muscle pulls on a bone affects the


mechanical effectiveness of the muscle-bone lever system. The force of muscular tension is
resolved into force components; 1 perpendicular to attached bone & 1 parallel to bone (only the
force acting perpendicular to bone [rotary component] causes bone to rotate about joint centre.
The component of muscle force directed parallel to the bones either away from joint centre
(dislocating component) or toward joint centre (stabilising component) depending on whether
the angle b/w bone & attached muscle is less or greater than 90°, e.g. elbow joint: max
mechanical advantage for brachialis, biceps & brachioradialis occur b/w angles at elbow
of approx 75-90°

165. List four things that can affect the joint torques produced from muscle contractions.
 Length of muscle affect contractile force
 Type of muscle pennate (parallel arrangement) affect contractile force
 Velocity of contraction
 Lever structure & Angle of Pull influences how much of the fibres are transferred to mechanical
work
 The angle of maximum mechanical advantage for any muscle is the angle at which the
maximum rotary force can be produced; as joint changes muscle length changes also – length
of muscle fibre at time of contraction influences force produced by the muscle.

CENTRE OF GRAVITY & STABILITY:


166. What is meant by the term ‘dynamic equilibrium’?
 Linear & angular acceleration occurring in motion of a body (accelerations are non-zero).
D’Alembert’s Principle: balance b/w applied forces & inertial forces for a body in motion.
Resulting motion is determined by evaluating the forces & moments based on acceleration of
object; not measuring forces directly.
 Linear Analysis on x-y components– ∑ Fx = max & ∑ Fy = may
 Dynamic Analysis - ∑Fx – max & ∑ T – I α (moment of inertia x angular acceleration). M = body
COM & a = acceleration of COM

167. The centre of gravity refers to what part of a body?


 COG is unique point about which a body’s weight is equally balanced in all directions; point as
which ∑ T (sum of torques) produced by the weights of the body segments is equal to 0. Sport:
lowering COG prior to take off enables lengthening of vertical path over which body is
accelerating during take off. Higher vertical velocity is important as take off velocity & angle
determines trajectory.

168. The COG is also synonymously referred to as what?

Downloaded by masoud masoudi (masoudmasoudi9777@gmail.com)


lOMoARcPSD|10025139

 Mass centroid & centre of mass (COM)

169. Where is the COG located in a perfectly symmetrical object?


 Geometric centre of object

170. Is it possible for the COG to be located outside an object and if so, provide an example?
 Yes, a hula-hoop/doughnut has its COG outside the object within the inner hole outside the
physical mass. Positions of segments may result in COG outside of body, e.g. body curling
around a bar during high jump/pole vault

171. The COG is used as a reference for analysis during which


types of motion?
 Linear Motion; gravity & all other forces acts through COG;
body behaves mechanically as though all its mass is located
at COG.

172. List TWO methods for determining the COG.


 Reaction Board Method: static posture determination;
mathematical calculation (rigid board
& scales); COG involves ∑ T about
support platform
 Segmentation Method: using x-y co-
ordinates after digitalisation. Use
cadaver studies, mathematical
modelling & radioisotopic scanning;
predicted parameters used to estimate mass & location of COM presented as a % of segment
length from prox to dist end of segment.

173. Define Stability.


 Resistance to disruption of equilibrium.

174. Define Balance.


 Ability to control equilibrium.

175. Explain the differences between stable, unstable and neutral


equilibrium and provide examples for each.
 Stable: if displacement is due to work done by the force & returns
to normal/original position, e.g. child sitting on a swing
 Unstable: If displacement is ↑; may continue to move away from
original position, e.g. ball rolling down hill
 Neutral: if displaced, stops & moves to a new position, e.g. ball
on flat surface with applied force

176. List the FOUR factors stability is dependent on?


 Where the line of gravity falls with respect to base of support: ↑
base of support , ↑ base of support in direction of oncoming force
& positioning line of COG closer to edge of base of support were
oncoming force applied ↑ stability, e.g. usage of walking frame
 The height of the COM relative to base of support: stability of
object is inversely proportional to height of COM. ↓COM =
more stable, e.g. lower COG in rugby union scrum.
 The Total Mass of the Object: stabiltiy is dependeant on
mass of object which may influence stabiltiy. ↑ mass = more
stability

Downloaded by masoud masoudi (masoudmasoudi9777@gmail.com)


lOMoARcPSD|10025139

 The Friction between surfaces: greater the friction b/w an object & surface it contacts the
greater the force requirement to initiate/maintain m/ment

177. Explain why a position of minimal stability is advantageous in swimming starts.


 Minimal stability; little energy needed to move→ quicker start/jump off blocks. Similar to runner;
forward lean.

178. When all other factors are held constant, what FIVE principles of stability could improve a
body’s ability to maintain equilibrium?
 ↑ body mass
 ↑ friction b/w body & surface/s contacted
 ↑ size of base of support in direction of line of
action of external force
 Horizontally positioning COG near edge of
base of support on oncoming external force
 Vertically position COG as low as possible

ANGULAR KINETICS:
179. Define Moment of Inertia; provide its
symbol/s and unit of measurement.
 Inertial property for rotating bodies
representing resistance to angular
acceleration; based on both mass & distance
the mass is distributed from the axis of rotation.
Every body is composed of particles of mass,
each with its own particular distance from a
given axis of rotation. The moment of inertia for a single particle of mass may be represented
as I = mr² - r = individual particles radius of rotation (from axis of rotation). For entire body; I =
mr². Symbol = I; measured in kg/m²

180. Which has greater affect on the moment of inertia of a body: The total mass or where the
mass is distributed relative to the axis of rotation?
 Moment of inertia ↑ with mass & distance. The further the mass is distributed from
axis of rotation that greater the moment of inertia. The mass distribution has greater
affect on moment of inertia than total mass. The more closely mass is distributed to
the axis of rotation, the easier it is to initiate/stop angular motion, e.g.
turntable/swivel chair – ↓ moment of inertia when mass is distributed closer to
centre of rotation [arms folded] & gymnastics; twist about longitudinal axis –
mass of gymnast distributed relatively close & along axis. However, if
gymnast rotates about transverse axis (forward/backward); the same mass is
distributed further from axis of rotation. Since there is a greater mass
distribution around transverse longitudinal axis, the I is greater due to greater
resistance to rotation to transverse axis.
Thus, may have to assume diff body
position; tuck – body mass closer to
transverse axis ↓ I – provides less
resistance to angular acceleration.

181. Why is it difficult to determine the moment of


inertia for the body?
 Moment of Inertia is not fixed (continuously
changes); body (muscle/fat) have diff densities
– uneven distribution (heterogenous). Can
approximate using data from cadaver studies,

Downloaded by masoud masoudi (masoudmasoudi9777@gmail.com)


lOMoARcPSD|10025139

mathematical modelling & photogrammetric methods.


 I = mk² - Moment of inertia = mass x radius of gyration²
 Parallel Axis Theorem: human body parts have been placed in a rotary pendulum to determine
their resistance to rotation. The procedure for determining moment of inertia of a no. of linked
segments is similar to segmentation method – IA = ICG + md². Must 1st determine the moment of
inertia for each segment using this formula then summing the individual moments of inertia
determines total.

182. What is the radius of gyration?


 Represents an object’s mass distribution with
respect to a given axis of rotation. Radius of
gyration (R or K) is always longer than radius
of rotation (based on r² of individual particles).

183. Is the radius of gyration the same point as the


COG for that segment?
 No

184. Explain, with respect to moment of inertia, how and why children’s sporting equipment varies
from that of adults.
 Due to children having a smaller distribution of mass due to smaller body segments, equipment
is modified and smaller in size, e.g. smaller baseball bat – automatically grip is lower on
bat handle; moment of inertia about wrist is ↓ thus makes it easier to swing. If using
adult-sized bat, would be too hard to swing, child would grip lower down on bat –
unneeded if already had appropriate equipment.

185. Explain which somatotypes (body shape and size) are suited to sports like gymnastics and
diving.
 Ectomorph (small build; slight muscular development) compared to mesomorph (well muscled,
wide shoulders, narrow waist) & endomorph (fat) have better inertial properties due to
appropriate smaller body segments with distribution of mass closer to centre of rotation
allowing quicker, more efficient turns/spins as required in gymnastics/diving.

186. Define Angular Momentum; provide its symbol/s and unit of measurement.
 Quantity of angular motion possessed by a body; measured as the product of moment of
inertia & angular velocity; H; kg/m²/s. Concepts of momentum are analogues; derived from
linear equivalent.
 H = I ω or H = mr² ω – I (moment of inertia) & ω (angular velocity)

187. Can an object have angular momentum without any angular velocity?
 No angular velocity in body = no angular momentum. As mass or angular velocity ↑, angular
momentum ↑ (mass distribution about axis of rotation has greatest effect on angular
momentum though radius of gyration.

188. What is meant by the principle of ‘Conservation of Momentum’?


 The total angular momentum of a given system remains constant in the absence of external
torques (when gravity is the only external force); as in projectile motion (angular momentum at
take off remains constant for duration of flight; magnitude & direction of angular momentum
vector for airborne performer is established at instant of takeoff) – momentum conserved
because body weight vector acting through total body COG creates no torque as moment arm
is 0.
 Even when angular momentum is 0, twist can be generated midair but requires skilful m/ment
of at least 2 segments: 1 part of body rotating 1 way & the lower body the other; they cancel
each other out but still rotate, i.e. k = 0, e.g. cat rotation : cat dropped always lands on 4
paws.

Downloaded by masoud masoudi (masoudmasoudi9777@gmail.com)


lOMoARcPSD|10025139

189. Explain how an athlete can transfer angular


momentum from one axis of rotation to another.
 Manipulation of moment of inertia & radius of
gyration – modified & trained technique, e.g. diver
somersault

190. If angular momentum cannot be gained or lost


once a body is rotating in the air, how then does a
diver increase their rate of somersaulting during a
dive?
 Diver in tuck position; ↓ Radius of gyration &
body’s moment of inertia ↓ in transverse axis; angular
momentum is constant with an ↑ in angular velocity
(tighter the tuck, the more angular velocity; distribution
of mass closer to COR). To spin faster/slower –
manipulate moment of inertia; take off large moment of
inertia & small angular velocity (rate of spin) as in lay
out position→ tuck position ↓ moment of inertia & ↑ in
angular velocity as quantity of angular momentum is
constant; before landing they open up or return to lay
out position to ↑ moment of inertia & slow down rate of spin (↓ angular velocity).

191. Define Angular Impulse.


 Change in angular momentum equal to the product of torque & time interval over
which the torque acts. When a net external torque acts; angular momentum of
system changes predictably (depends on magnitude & direction of external
torques & length of time); when angular impulse acts there is a change in the
total angular momentum of the system.
 Angular Impulse = Tt (Torque x time) – Force x moment arm x time

192. Explain the link between angular impulse and angular momentum.
 As angular impulse is the product of torque & time, significant changes in an
object’s angular momentum may result from the action of a large torque over a
small time interval or small torque over a long time. Since torque is product of a
force’s magnitude & perpendicular distance to axis of rotation, both factors affect
angular impulse.
 Sport: throwing events; arm maximise angular impulse exerted on the implement prior to
release→ maximisation of momentum, thus ↑ horizontal displacement. When moment of inertia
is constant; ↑ angular velocity ↑ tangential velocity.
 Hammer Throw: 3 or 4 body turns with slight trunk flexion allows further reach with hands (use
hips to counter the centrifugal force); ↑ radius of rotation ↑ moment of inertia – if angular
velocity is maintained then angular momentum will ↑ thus ↑ distance. Aerial rotation: motions of
body segments during take off determine magnitude & direction of reaction force which
generate both linear & angular impulses.

193. Define Angular Work, and provide its unit of measurement.


 Magnitude of torque against an object & angular distance the object rotates
in direction of applied torque.
 Angular Work = T∆Ө Torque x change in angular distance (∆Ө); Nm or J,
e.g. if a 40.5 Nm torque is applied over a rotation of 0.79 rad, work
done is: Work = 40.5 x 0.79 rad

Downloaded by masoud masoudi (masoudmasoudi9777@gmail.com)


lOMoARcPSD|10025139

194. The angular work done by a muscle is either positive or negative. Differentiate between the
two and indicate the muscle contraction types associated with each.
 Muscle contracts & produces tension to move segment; a torque is produced at joint &
segment moves through angular displacement
 POSITIVE: concentric muscle action (shortening) as tension created, e.g. weightlifter –
barbell bicep curl; flex at elbow (concentric pause) bringing barbell up; torque due to
biceps greater torque due to barbell. Greater metabolic expenditure than NEG.
 NEGATIVE: eccentric muscle action (lengthening) as tension created, lowering barbell in bicep
curl; resisting pull of gravity – the flexor muscle does negative work (barbell does work on
muscle); torque due to biceps less than torque due to barbell.

 Rotational Kinetic Energy: RKE = ½ I ω (rotational kinetic energy = ½ x moment of inertia x


ang vel)
 Total Mechanical Energy: TE = KE + PE + RKE (Rotational kinetic energy)

195. Define Angular Power, and provide its unit of measurement.


 The product of torque & angular velocity; watts or Nm/s. Used to describe mechanical muscle
power.
 Angular Power = T ω
 POSITIVE muscle power: POSITIVE work or net concentric
action
 NEGATIVE muscle power: NEGATIVE work of net eccentric
action

196. The Analogue of Newton’s 1st Law of Motion states that …...
 An object will continue spinning with constant angular momentum unless acted on by an
external torque.

197. Give an example of the analogue of Newton’s 1 st law of motion.


 Divers & gymnasts able to achieve aerial manoeuvres by manipulation of moment of inertia &
angular velocity due to constant angular momentum (remains constant due to absence of
external torque)

198. The Analogue of Newton’s 2nd Law of Motion states that …...
 A net torque produces angular acceleration in direct proportion to the magnitude of the torque
& in the same direction as the torque & angular acceleration is inversely proportional to the
body’s moment of inertia. T (External Torque) = I α (moment of inertia x angular acceleration)

199. Give an example of the analogue of Newton’s 2 nd law of motion.


 If person abducts arm from body to horizontal position; torque at shoulder results in angular
acceleration of arm – greater moment of inertia of moment arm about axis through shoulder &
less angular acceleration of segment.

200. The Analogue of Newton’s 3rd Law of Motion states that …...
 For every torque exerted by 1 body on another, there is an equal & opposite torque exerted by
the 2nd body on the 1st; torque produced by 1 body part to rotate that part results in counter
torque by another body part

201. Give an example of the analogue of Newton’s 3 rd law of motion.


 Counter-Rotation Effect: Rotation can be initiated in the air when angular momentum = 0, e.g.
m/ment of hitting arm with high angular velocity, there is a compensatory rotation of
lower body producing an equal angular momentum in opposite direction.

CENTRIPETAL AND CENTRIFUGAL FORCES:

Downloaded by masoud masoudi (masoudmasoudi9777@gmail.com)


lOMoARcPSD|10025139

202. Define Centripetal Force.


 Force directed toward the centre of rotation for a body in rotational motion; force which
prevents a whirling object from leaving its circular path.

203. Centripetal forces always act in which direction?


 Always directed towards centre or rotation of the rotating
object.

204. What is an alternative name given to centripetal force?


 Centre Seeking Force

205. Centripetal forces are associated with which acceleration?


 Radial component of angular acceleration of body travelling around a
curved path.

206. The magnitude of centripetal force is most influenced by which property?


 Angular Velocity
 Cycling: ground exerts centripetal force on tires. Forces acting on cycle-
cyclist system are weight, friction & GRF.

207. Define Centrifugal Force.


 Reaction force to centripetal force (Newton’s 3rd Law); equal in magnitude but opposite
direction compared to Fc. Acts against wire, chain etc.

208. Centrifugal forces always act in which direction?


 Always directed away from centre of rotation of the rotating object, e.g. sensation of being
pushed in direction of outside curve in vehicle going around corner (in actual fact;
Newtons 1st law of body’s inertia tends to cause it to continue travelling straight, rather
than a curved path; the car seat, seat belt & car door actually change direction of body
motion. More appropriately, the absence of centripetal force acting on an object.

209. What is an alternative name given to centrifugal force?


 Centre Fleeing Force

210. Can either centripetal or centrifugal force exist without the other?
 No (can’t have 1 without the other)
 Running: If both centripetal & centrifugal forces did not exist; runner would run in straight line –
needed in order to run around track.

THE GAIT CYCLE:


211. Gait is divided into which two phases?
 Stance Phase: foot in contact
 Swing/Flight Phase: no contact

212. What percentage is spent in each gait phase during


walking?
 Stance (60%) & Swing (40%)

213. What percentage is spent in each gait phase


during running?
 Stance (45-70%) & Swing (32-50%); b/w jogging to
sprinting

Downloaded by masoud masoudi (masoudmasoudi9777@gmail.com)


lOMoARcPSD|10025139

214. Define each of the following terms in relation to gait:


i. Stance: foot in contact; the period of foot ground contact
ii. Flight / Swing: no contact; period of no foot ground contact
iii. Single Support: one foot contacting ground
iv. Double Support: both feet in contact with ground
v. Initial Contact / Heel strike: first touch of heel on ground
vi. Heel Lift (HL): lifting of heel off ground
vii. Toe Off (TO): When last part of foot leaves ground
viii. Flat Foot / Fore Foot Loading (FFL): loading of weight; force on whole foot
surface
ix. Step: Left heel strike to right heel strike
x. Stride: Left heel strike to left heel strike
xi. Cadence / Frequency/Rate: no. of steps per minute

215. Walking is defined by what?


 There is always contact with ground by 1 foot or both feet; there are 2
double support periods (initial & final 10% of stance phase)

216. Running is defined by what?


 Has an airborne phase known as double float; times
when neither foot’s on the ground. Occurs at beginning &
end of swing phase; in running the swing phase is larger
than in walking to allow for the double floats.

217. Draw and label VGRF plots for both walking and
running and indicate any points (peaks) of interest.
 Walking: double-bump (bi-modal); 1st peak (passive/impact peak force due to heel strike; when
the body is lowered under eccentric muscular control – Heel Striker Runner [Mid-Foot Strike
Runner has little or no impact peak]) & 2nd peak (active peak force due to toe off; as foot
prepares for take off; force development by muscle’s to accelerate body off ground); drop in
b/w 2 peaks (mid-stance/knee back)
 Running: Initial small impact force peak followed by gradual larger active peak (uni-modal).
 Anterior-Posterior GRF (forward/backward): similar shape for walking & running; magnitude
b/w walking (0.15BW) & running (0.5BW). During locomotion, A-P component: NEGATIVE
PHASE (during 1st half of support [bracing impulse=slowing]) & POSTIVE PHASE (during 2nd
half of support [acceleration impulse=speed up])
 Medio-Lateral GRF: variable with no consistent pattern b/w subjects; depends on foot
placement; foot adduction (toeing in) & foot abduction (toeing out) – inversion/eversion may
affect GRF; magnitude b/w 0.01 -0.1BW.

218. What factors can influence the GRF’s in running?


 Running speed, running duration, knee flexion angle at contact, stride length, fatigue, footwear,
surface stiffness/smoothness, light intensity & inclination (grade).
 Running style will influence GRF’s. Magnitude of GRF contact varies with diff speeds (↑ in
relation to speed), will ↑ with ↑ distance from which individual lands (VGRF ↑ when landing
from higher height; impulse momentum relationship to landing) & HGRF ↑ when overstriding.
VGRF always has greatest magnitude.
 Vertical Component: stress on bones/muscles during a jumping action
 Horizontal Component: motion resulting for stride length & frequency

219. Define each of the factors that directly influence walking & running speed.

Downloaded by masoud masoudi (masoudmasoudi9777@gmail.com)


lOMoARcPSD|10025139

 Running/Walking Speed: Speed= stride length (displacement) x stride rate (time)→ S=DxT.
Thus, ↑ velocity by ↑ stride length, rate & both.

220. How can a runner increase their running speed?


 ↑ stride length/stride rate
 Running: Velocity up to 7m/s (↑ in stride length/rate are
linear). At higher velocity, smaller ↑ in stride length &
greater ↑ in stride rate. During sprinting, a runner ↑ velocity
by stride rate not length.
 When ↑ velocity from a slow pace the ↑ is due to ↑ stride
length but for further ↑, recreational runners rely more on ↑
stride frequency. Elite runners prefer a given stride
frequency over a range of slow to moderate running speed;
the most efficient stride frequency. Most runners
automatically choose combo of stride length/frequency that minimises physiological cost of
running. Avoid long stride length (injury); over-striding=↓ in velocity due to ↑ vertical
displacement; Newton’s 3rd law; greater opposite force when over-striding→ slow down. Max
running speeds usually reached early in race but many lose speed close to end. However,
some may run at a constant pace & achieve max at the end.

221. Stride length is a function of what?


 Stride: heel contact of R to subsequent heel contact on R. Divided into stance (support) or
swing (non-support phase). Stride Length: distance covered by 1 stride. Function of body
height (taller-longer stride), muscle length (more flexibility, ROM- ↑ frequency), footwear, state
of fatigue, injury history, inclination of surface (hill-shorter stride), clothes, wind, environment
(sand/cement). Stride Time: time taken for a single stride.
 Step: foot contact on R to foot contact on L. Step Length: distance covered by 1 step/ Step
Length: no. of strides/min.

222. Why should over-striding be minimised?


 Longer strides tend to generate GRF’s with larger retarding horizontal components; thus
overstriding is counterproductive.

223. What is Pace in relation to running gait?


 Pace is often used rather than speed/velocity when analysing races. Pace is inverse of speed,
e.g. time taken to cover a distance, such as min per km/sec per 400m.

224. Explain how / why velocity changes during ground contact when walking and running.
 Velocity changes during ground contact. Horizontal Vel ↓ at foot contact/beginning of stance.
Continues to ↓ until leg extends; leg extension drive ↑ vel (rearward force application). Acc-time
graph during stance shows both + & - acc.

225. What must a person do to ensure a constant velocity when walking or running?
 To maintain constant VelAV (running speed); manage/manipulate loss & gain in velocity through
finding balance b/w stride length/stride rate (loss in velocity = gain in velocity – Change (∆) in
acceleration = 0)

FLUID MECHANICS:
226. What is a Fluid?
 Any substance that tends to flow or continuously deform when acted on by a shear force, e.g.
liquid, gas, air, water

227. Explain what is meant by the term ‘Relative Velocity’.

Downloaded by masoud masoudi (masoudmasoudi9777@gmail.com)


lOMoARcPSD|10025139

 The velocity of a body in a fluid depends on the motion of the body & fluid; relative velocity is
usually relative to the fluid but occasionally relative to another object. Velocity relative to the
fluid influences the forces acting on the object.
 When a body moves through a fluid, the fluid motion alters (waves, currents); when velocity
relative to the fluid is slow there is laminar flow of the adjacent or surrounding fluid. Relative
velocity with respect to fluid is to the vector subtraction of velocity of wind from velocity of body
 Tail Wind: same direction; velocity relative to
wind ↓
 Head Wind: opposite direction; velocity
relative to wind ↑
 Relative Motion: body moving through a
stationary fluid (swimming) & fluid moving
past a stationary body (wind tunnel).

228. Define Laminar Flow.


 Flow characterised by smooth, parallel layers of fluid; the pathways of fluid
particles don’t cross.

229. Explain what Turbulence is and how it is created.


 Flow characterised by mixing of adjacent fluid layers. As velocity relative to
fluid ↑, flow of fluid becomes turbulent (random/swirling); if turbulence is present flow is non-
laminar, e.g. turbulent flow under hand/drag over hand

230. Define Density; provide its symbol/s and unit of measurement.


 Mass per unit of volume; ῤ (rho); kg/m-3
 ῤ = mass/volume

231. List in order of most dense to least dense the following substances:
Muscle, Water, Fat, Air and Bone.
 Muscle: 1030-1090
 Water: 1000
 Fat: 950-990
 Air: 1.21
 Bone: 1100-1800

232. Define Specific Weight; provide its symbol/s and unit of measurement.
 Weight per unit of volume; ɣ (gamma); Nm-3
 ɣ = weight/volume

233. What affect would increasing the density and specific weight of a fluid have on the force
exerted by that fluid on another body?
 ↑ density & specific weight of fluid ↑ magnitude of force exerted by the fluid of the object, e.g.
golf ball & table tennis ball have approx same volume but golf ball greater
density/specific weight

234. Viscosity refers to what?


 Measure of a fluid’s internal resistance to flow

235. Increasing the viscosity of a fluid would result in what occurring?


 ↑ forces by fluid on object; not related to density

236. In what ways do temperature and pressure influence the density and viscosity of fluids?
 ↑ Temp = ↓ Density & ↓ Viscosity
 ↑ Pressure = ↑ Density & ↑ Viscosity

Downloaded by masoud masoudi (masoudmasoudi9777@gmail.com)


lOMoARcPSD|10025139

237. Define Buoyancy.


 Fluid force that acts vertically upward; scalar.

238. Archimedes’ Principle states that…


 The buoyant force acting on an object in a fluid is equal to the weight of the fluid displaced by
the body. The body which is partially/fully immersed in a fluid will experience an upward
buoyant force
 Fb = Vd ɣ - (Buoyant force) = Volume displaced x specific weight of fluid

239. In which direction do buoyant forces always act?


 Buoyant force is always directed upwards. Buoyancy pushes body up
until out of water/will stop when force equals body weight. Max buoyant
force when the body is totally submerged (max volume or water
displaced); when totally submerged – volume of water displaced =
volume of body

240. What affect does increasing the density of a fluid have on the buoyant
force that fluid is capable of exerting?
 ↑ density of fluid = ↑ Fb.

241. The Centre of Volume refers to what?


 Point about which a body’s volume is equally balanced, through which
the buoyant force acts.

242. What determines whether a body will float or not?


 Flotation depends on r/ship b/w buoyancy & weight. When Fb & W are the only 2 forces; Fb =
W (magnitude of buoyant force & body weight are equal) will have equilibrium (static; when
COV & COG aligned vertically [if not equal; sink as this causes force couple – causing rotation;
to counteract this we extend arms & bend legs to elevate CG & move it closer to COV])→ float;
W > Fb = sink.
 Density = Mass/Volume: large mass/small volume = high density & small mass/large volume =
low density (more likely to float)
 Body Composition: density of muscle/bone > fat (fat people more likely to float as fat is less
dense)
 Lung Volume (air; less dense than water); more air in lungs allows better floating.

243. For most swimmers, floating occurs when?


 Most bodies float statically in a partially submerged position; when buoyant force is larger than
the body’s weight. Sinking = buoyant force is smaller than body’s weight

244. Why don’t most people float level with their body parallel to the surface of the water?
 CG inferior to centre of volume due to relatively low eight & high volume of
lungs; feet sink as a torque is generated→ sink until weight force & Fb are
both vertical & in the same line.

245. What is Drag and in which direction does it usually


act?
 Force caused by dynamic action of fluid; frictional force.
Usually resists m/ment of an object through a fluid. Acts
in direction of the free-stream fluid flow. Fd. Frontal drag for skater/cyclist
less than vertical drag of parachutist (large area; large drag); although
main component is relative velocity.

Downloaded by masoud masoudi (masoudmasoudi9777@gmail.com)


lOMoARcPSD|10025139

246. From the formula for drag, list and explain the affect these four factors have on the magnitude
of drag force.
 ↑ Fluid Density→ ↑ Fd
 ↑ Coefficient of Drag→ ↑ Fd
 ↑ Velocity relative to fluid→ ↑ Fd
 ↑ Projected area of the body or surface area of body perpendicular to fluid flow→ ↑ Fd

247. What affect would high altitude have on total drag?


 Air density ↓ with ↑ altitude; thus ↓ drag force - ↑ performance due to ↑ relative velocity & ↓
resistance against object, e.g. run faster

248. Name the THREE components of drag.


 Surface Drag
 Form Drag
 Wave Drag

249. What two other names are often given to Surface Drag?
 Surface Friction/Viscous Drag/Skin Friction

250. Define and explain Surface Drag.


 Resistance derived from friction b/w adjacent layers of fluid
near a body moving through fluid. Particles in immediate
contact slowed due to shear stress exerted by body on
fluid; next layer slightly slowing due to friction b/w adjacent layers→ effect continues onwards.
No. of layers affected ↑ with ↑ velocity, ↑ surfaces areas of body, ↑ roughness & ↑ viscosity.
Boundary Layer is volume of fluid where volume is diminished; laminar boundary layer
becomes thicker as flow passes along plate. Surface friction is the major portion of drag
present when flow in primarily laminar

251. Explain how surface drag is minimised in some sports.


 ↓ friction by altering clothing to smoother/tighter fitting (swimming suit/cap) or shaving body hair
(cycling/sprinting) or using lubricant oil on body.

252. What two other names are often given to Form Drag?
 Profile/Pressure Drag

253. Define and explain Form Drag.


 Resistance created by a pressure differential b/w the lead
& rear sides of the body moving through a fluid. When boundary layer is primarily turbulent;
form drag predominates – major contributor to drag during most human & projectile motion.
When a body moves fast enough to cause a pocket of turbulence, there is an imbalance in
pressure surrounding the body. Upstream = high pressure (HP) & Downstream = low pressure
(LP); whenever a pressure differential exists there is a force going from HP to LP region.

254. The magnitude of form drag is dependent on what 3 things?


 Relative Velocity
 Magnitude of pressure gradient
 Surface area perpendicular to flow (cross-sectional area)

255. Explain how Streamlining can minimise form drag and provide 2 examples.
 Pressure gradient & surface area perpendicular to flow can be minimised to ↓ the effect of form
drag by streamlining; ↓ cross-sectional area which ↓ turbulence & thus ↓ drag. E.g. cycling
helmets, wet suit (buoyancy to ↓ drag) & golf balls (slightly rough surface; have dimples
to ↓ drag as there is less contact of air to surface).

Downloaded by masoud masoudi (masoudmasoudi9777@gmail.com)


lOMoARcPSD|10025139

256. How does the tear drop shape of a track cycling helmet decrease from drag?
 Streamlined shape ↓ turbulence created at the trailing edge of a body in a fluid (less difference
b/w HP & LW); ↓ drag

257. How do the dimples on a golf ball decrease from drag?


 Produce a turbulent boundary layer at the ball’s surface that ↓ form drag due to ↓ turbulence at
trailing edge of golf ball on the ball over the range of velocities at which a golf ball travels. The
dimples separate the air-flow from the ball creating more molecular impacts at rear of the ball;
↓ the difference b/w pressure at the front & back as the trailing turbulence curves around &
push the ball from behind.

258. Explain how Wave Drag is generated.


 Resistance created by the generation of waves at the interface b/w 2 diff fluids such as air &
water. Acts as the interference b/w 2 diff fluids. Has less affect for fully submerges objects that
don’t break interface b/w 2 surfaces – major contributor when swimming. As a body segment is
moved along, near or across the air/water interface, a wave is created in the more dense fluid.
The reaction force of water on body constitutes wave drag. Wave drag ↑ with greater up &
down motion of body & with ↑ speed.

259. Explain how wave drag can be minimised.


 Skilled swimmers produce a smaller bow wave than unskilled swimmers at same speed
presumable due to less up & down motion in better technique. At very fast swimming speeds,
wave drag is generally the largest component of total drag acting on a swimmer.

260. What are Lift Forces and in which direction do they act?
 A force acting on a body in a fluid that acts in a direction perpendicular to the fluid flow; actin
any direction (not just up) – affected by same factors as drag (just substitute Fl with Fd & Cd
with Cl [based on shape]).

261. Bernoulli’s Principle states that …


 Regions of relative high velocity fluid are associated with regions of
relative LP; regions of relative low velocity fluid are associated with
regions of relative HP. Fluid flowing over curved surface accelerates; ↑
velocity flow & result in lift force – difference in velocity = a pressure
difference.

262. How do foil shapes provide lift forces?


 A foil is a shape capable of generating lift in the presence of fluid flow,
e.g. aeroplane wings. When a fluid meets a foil, the fluid
separates; some moving straight back along the flat bottom
surface & fluid over the curved (top) surfaces is POSITIVELY
accelerated (region of relatively high velocity). When HP & LP
on opposite sides of foil; life force perpendicular to the foil
directed towards the region of LP (force is always directed in
direction of LP zone).

263. How can lift force be increased?


 Greater velocity of force relative to fluid ↑ lift. ↑ fluid density ↑
lift. ↑ surface area of flat side of foil ↑ list
 Swimming: Swimmers hand creates a foil as slices through
water, lift is generated.

264. Define Angle of Attack.

Downloaded by masoud masoudi (masoudmasoudi9777@gmail.com)


lOMoARcPSD|10025139

 Angle b/w longitudinal axis of a body & direction of fluid flow; important in launching a lift
producing projectile for max range (balance of lift & drag depends on size, shape & material of
object). A POSITIVE angle of attack required to generate a lift force. As angle of attack ↑ then
surface area to fluid flow ↑ = ↑ drag. Angle too steep = fluid can’t flow along curves surface; no
lift created.

265. Explain the Lift / Drag Ratio.


 Optimum angle of attack for maximising range is the angle at which lift/drag ratio is maximum.
If drag & lift are too small, angle of attack doesn’t create a sufficiently HP across top & bottom
surfaces of the foil. When the angle of attack is too large, the fluid cannot flow over the curved
surface of the foil & no lift is generated. When angle of attack is below horizontal, lift is created
in a downward rotation. E.g. ski jump; 1st part of flight (assume small angle of attack to
minimise drag) & 2nd part (↑ angle of attack up to that of max lift [drag])

266. What is a Magnus Force and under what conditions can it be observed?
 Lift force generated by spin. When object spins
in a fluid on 1 side; collide with molecules in fluid
free stream causing a region of relative high
velocity & LP. Magnus force is the pressure
difference; lift force directed from HP to LP
region; usually has a deviated path.
 Spin & its effect on pressure zones: HV=LP (not
as much air molecules colliding) & LV+HP
(Bernoulli’s Principle)

267. Which motion results from a Magnus force being applied to a ball?
 Magnus Effect: deviation in the trajectory of a spinning object toward the direction of spin
resulting from the Magnus force.

268. List TWO ways that the Magnus force experienced by an object can be increased.
 Topspin: When hit will go away from person; HP on top (ball slows down due to oncoming floe;
drops faster than if ball not spinning
 Backspin: Ball spins toward person after kicking. Will flow toward force or LP at top; stays
longer in air than if not spinning.
 Side Spin: sideward motion to ball; trajectory will follow as regular curve due to magnus effect.
Optical illusion causes batter to make judgement error
 It could be said that Magnus refers to spin & Bernoulli refers to swing in terms of ball m/ment,
e.g. cricket ball – rough side (air moves through quicker); balls swings towards rough
side due to LP (HP on other [smooth] side).
 Whirlpool (direction of ball spin [top/back spin] determines direction of lift force). Top Spin = ↓
lift force. Back Spin = ↑ lift

269. Name and explain two theories on how swimmers propel themselves through water
 Resistance drag acts on swimmer but propulsive forces exerted by the water in reaction to
swimmers m/ment→ forward motion through water.
 Propulsive Drag Theory: as swimmers hands & arms move backwards through water, the
forward directed reaction force produced propulsion (Newton’s 3rd law)
 Propulsive Lift Theory: swimmers use a foil shape of hand; rapid lateral m/ments through water
generates lift.
 Propulsion in a fluid medium: Headwind slows & Tailwind helps (tailwind velocity greater→
drag force in direction of motion→ propulsive drag)

HUMAN TISSUE BIOMECHANICS:


270. The effect forces have on the body depend on what 3 things?

Downloaded by masoud masoudi (masoudmasoudi9777@gmail.com)


lOMoARcPSD|10025139

 Direction
 Duration
 Magnitude

271. Define / explain each of the following:


i. Compression: squeezing force; pressing directly through body,
e.g. vertebra in spinal column. Compressive strength is ability
to resist a compressive force
ii. Tension: opposite of compressive force; pulling force creating
tension in object applied to, e.g. all muscles pull on attached
bone. Tensile strength is ability to resist tension
iii. Shear: force applied along longitudinal axis of bone, acts parallel or at tangent to surface –
slides/glides/displaces, e.g. knee joint
iv. Mechanical Stress: distribution of force within a body. Results from force distribution inside a
solid body due to application of force. Different to pressure which is distribution of force external to
solid body. Quantified as force per unit area over which force acts, e.g. lumber region – L5-S1
inter-vertebral disc leading to herniation.
v. Torsion: structure is caused to twist – longitudinal axis when 1 end of structure is fixed, e.g.
tibial fractures when position of foot is fixed while rest of body is twisted in football
vi. Bending: asymmetric loading producing tension on 1 side & compression on other side of body
in longitudinal axial.
vii. Combined Loading: more than loading present; human body subjected to simultaneous forces
in most daily activities.

272. What substances constitute the major building blocks of bone?


 Calcium carbonate & calcium phosphate (60-70% of dry bone weight)
 Collagen
 Water (25-30% of total bone weight)

273. What substance gives bones their stiffness and compressive strength?
 Minerals give bone its stiffness & compressive strength.

274. What substance gives bones their flexibility and tensile strength?
 Collagen gives tensile strength, resists tension & provides flexibility.

275. How porous bones are is dependent on what?


 Depends on mineral content (↓ mineral = more porous). Porous (miniature gaps in b/w bones
allowing minerals to circulate throughout).

276. When classifying bone based on porosity, low porosity and high porosity bones are
respectively named as what?
 Low Porosity: Cortical – shafts of long bones
 High Porosity: Trabecular – ends of long bones & in vertebrae

277. Describe the differences between cortical and trabecular bone in relation to their mechanical
properties.
 Cortical: stiffer & can withstand greater stress but less strain/deformity
 Trabecular: undergo deformity before fracturing, i.e. a break not strain

278. When is bone strongest?


 Bone is strongest against with compressive strength (of concrete) & has tensile strength (of
steel).

279. When is bone weakest?

Downloaded by masoud masoudi (masoudmasoudi9777@gmail.com)


lOMoARcPSD|10025139

 Bone is weakest against shear stress. Bones are not good under shear forces

280. Old Mother Hubbard went to the cupboard to fetch poor Rover a what?
 Old Mother Hubbard went to the cupboard to fetch poor Rover a bone. When she bent over,
Rover came over, and gave her a bone of his own

281. Under which forces are bones less likely to maintain their structural integrity?
 Bone can handle great load in longitudinal direction, but least (may lose structural
integrity) when load is applied across surface of bone.

282. The presence or absence of force on bones can result in what occurring?
 Presence or absence of force results in changes in bone size, shape & density. Forces
include BW & PA; diet, lifestyle & genetics. Hypertrophy; when forces ↑. Atrophy; when
mechanical stress ↓.

283. Which cell types are responsible for building and reabsorbing bone, respectively?
 Osteoblasts: build new bone
 Osteoclasts: reabsorb bone

 Functions of Skeletal System: leverage (levers & axes of rotation about which muscular system
generates m/ment; levers magnify force/speed; bones are levers & axes are where bone
articulates), support (provide support structure to maintain posture & accommodate for large
external forces), protection (internal organs), storage (fat & minerals) & blood cell formation.
 Joints – Type of Bone: axial (central), short (cubical; gliding motion), flat (protect organs/soft
tissue), irregular (diff shapes for protection), long (framework of skeleton) & sesamoid (alter
angle of insertion of muscle).
 Classification of Joint: Synarthoroses (immovable), amphiathroses (slightly movable) &
diarthrosis/synovial (freely movable)

284. Explain the role of Articular Cartilage.


 ↓ friction & wear; deforms under load & spreads load at joint over a wide area→ ↓ stress at
bone on bone contact by up to 50%

285. Explain the role of Articular Fibrocartilage


 Fibrocartilaginous discs or partial discs; distribution of loads over joint surface, improvement of
fit of articulating surfaces, limit slip/translation of 1 bone with respect to another, protect the
periphery of articulation, lubrication & shock absorption, e.g. meniscus of knee &
intervertebral discs

286. What affect has exercise shown on Articular Connective Tissue such as Tendons and
Ligaments?
 Exercise can ↑ strength & size, e.g. size of ACL is proportionate to strength of quadriceps.

287. Define Joint Stability.


 The ability of a joint to resist abnormal displacement of the articulating bones, i.e. fit tightly
together.

288. Joint stability is influenced by what?


 Influenced by shape of articulating surfaces & arrangements of ligaments, muscles & tendons.
 Close Packed Position: area of contact max ↑ stability
 Loose Packed Position: ↓ in contact area ↓ stability (disuse/being over stretched ↓ stability).

289. What effect do fatigued muscles have on joint stability and why?

Downloaded by masoud masoudi (masoudmasoudi9777@gmail.com)


lOMoARcPSD|10025139

 When muscles are fatigued, they contribute less to joint stability although muscle contraction
doesn’t always enhance joint stability. The force by a contracted muscle can be resolved into:
rotary component (perpendicular to attached bone) & component parallel to bone. If parallel
component is in direction of joint then muscle contraction is stabilising, but if it’s away from join
it’s dislocating.

290. Explain joint Flexibility and Range of Movement.


 Joint flexibility = ROM of joint. ROM is angle through which a joint moves from anatomical
position to the extreme limit of segment motion in a particular direction. Dependant on shape of
articular surfaces, fluid content of joint, intervening muscle, fat laxity, muscle elasticity &
collagenous tissue; joint specific.

291. What 4 behavioural properties do muscles exhibit?


 Extensibility: ability to lengthen
 Elasticity: ability to return to normal length after extension/contraction
o Parallel Elastic Component (PEC): passive elastic properties due to muscle membranes
o Series Elastic Component (SEC): passive elastic properties due to tendons (C/T ends)
o Contractile Component: active due to muscle fibres
 Irritability: ability to respond to stimulus & to maintain tension
 Ability to develop tension

 Role of muscles: agonist (causes m/ment), antagonist (opposes m/ment caused by agonist),
stabiliser (acts to reduce m/ment/stabilise a body part against force from another source) &
neutraliser (acts to eliminate an unwanted action of an agonist).

292. Explain each of the following muscle contraction types:

i. Concentric: shortening of muscle


ii. Eccentric: lengthening of muscle
iii. Isometric: no change in length of muscle
iv. Isokinetic: same speed (both concentric & eccentric)
v. Isotonic: same force (both concentric & eccentric)

293. What biomechanical considerations are affected by muscle characteristics when muscle
tension is developed?
 Magnitude of generated force
 Speed of force development
 Length of time force is maintained

294. What is Muscle Tetanus and the affect caused if it is prolonged?


 State of muscle producing sustained maximal tension resulting from repetitive stimulation.
Rapid, successive volley of stimuli; summation occurs & tension ↑ until max is maintained for a
time then fibre is in tetanus. If tetanus is prolonged→ fatigue causing gradual ↓ in tension.

295. What is a Motor Unit and what role do they play in muscle contraction?
 Muscles made up of motor units; smallest subunit of muscle that is under neutral control. Motor
unit consists of a synaptic junction in the ventral root of the spinal cord, a motor axon, a motor
end plate in muscle fibres & muscle fibres it innervates. 1 motor unit may innervate anywhere
from 3-2000 muscle fibres depending on what the muscle needs. ↑ muscle fibres innervated ↑
force; fine/gross m/ment require ↓ fibres innervated. Single muscle made up of groups of
muscle fibres; organised in groups innervated by a single motor neuron called motor units.

296. Which muscle fibre types are innervated first during contraction and why?
 Slow-twitch muscle fibres are generally innervated by motor units with the lowest functional
threshold & are thus 1st recruited even when motion is fast. Fast-twitch fibres are innervated by

Downloaded by masoud masoudi (masoudmasoudi9777@gmail.com)


lOMoARcPSD|10025139

higher threshold motor units & are activated during rapid/powerful m/ment. Depending on
force; speed &/or duration required determines twitch fibre used.

297. Explain several differences between Slow Twitch and Fast Twitch muscle fibres.
 Slow Twitch (ST) Fibres (Type I): ↑ capillarisation (receives larger blood supply), ↑ resistance
to fatigue, ↓ rate of work & ↑ training
 Fast Twitch (FT) Fibres (Type IIa & IIb): ↓ resistance to fatigue, ↑ rate of tension development
& ↑ contraction force (more powerful)

298. List the two main types of muscle fibre arrangement and different effects they produce.
 Parallel: large ROM; occurs around a joint. Tendon on either end, 1 muscle belly & parallel to
line of tendon
 Pennate: large forces; tendon through muscle, at an angle of line of tendon into bone & ↑
fibres. As fibres contract angle of pennation ↑, when at greater than 60° - force transferred
through tendon is less than half the force actually applied by fibres; this ↓ occurs because it is
only the component of force in the direction of tendon that pulls on the bone, but arrangement
enables more fibres per unit volume, thus overall force ↑.

299. What are Tendons and what do they do?


 Attach muscle to bone & consist of highly organised bundles of collagenous fibres; both
passive tissue (slightly extensible [stretchable under tensile force]) & elastic (low elastic limit).
Parallel fibres give structural rigidity to tendon; usually covered in sheath with a surrounding
lubricating fluid. Tendons can withstand high tensile force. Exhibit viscoelastic behaviour in
response to loading; e.g. Achilles tendon can resists tensile loads to a degree matching
or greater than steel.

300. Tendons and muscles are joined at the what?


 Tendon & muscle join at myotendinous junction; together
absorb/generate tension.

301. Explain the Force – Velocity Relationship.


 When the resistance (force) is small, muscle contracts with
maximal velocity. As the load progressively ↑, concentric contraction
velocity slows to 0 at isometric maximum. As the load ↑ further, the
muscle lengthens eccentrically; force-velocity curve
 For concentric contraction: tension (force) in a muscle ↓ as
velocity of contraction is ↑
 For eccentric contraction: tension in a muscle ↑ as the
velocity of contraction is ↑

302. Explain the Force – Length Relationship.


 In an isometric contraction Fmax is dependent partially on muscle length. Force generation
peaks when muscle at normal resting level (not stretched or contracted); however in humans,
force generation ↑ with slight muscle stretch.

303. The Force – Time Relationship indicates what?


 There is a delay b/w neutral stimulation & development of tension within muscle.. Fmax may
take a further 1 second (lower with fast twitch muscle fibres)

304. What is Electromechanical Delay (EMD)?


 Time b/w arrival of neutral stimulus & tension development of muscle.

305. What effect do muscle length, contraction type, contraction velocity and fatigue have on
EMD?
 No effect

Downloaded by masoud masoudi (masoudmasoudi9777@gmail.com)


lOMoARcPSD|10025139

306. What does a high % of FT fibres and / or training have on the time taken to develop maximal
force in a muscle?
 Shorter max force development times are associated with a high % of FT fibres & in a trained
state.

307. Define Muscular Strength.


 Ability of a given muscle group to generate torque at a particular joint.

308. The Torque generated at a joint by a muscle is dependent on what?


 Max tension developed in the muscle
 Location/Orientation of muscle with respect to joint centre

309. Tension / Force development in a muscle is dependent on what?


 Muscle cross sectional area; more fibres more force
 Training State (Neural Adaptation); FT & ST not different in force generating per unit cross
sectional area
 Angle of tension to bone; distance b/w attachment & joint centre affects moment arm length.

310. Muscle Power is equal to what?


 Product of force & velocity. P = fv; units are W or Nm/s.

311. The production of maximum muscle power is dependent on what?


 Max P occurs at 1/3 max velocity which equals 1/3 max concentric force. Power ↑ with ↑
velocity only occur up to a certain point. Given the force-velocity relationship of ↓ force as
velocity of application ↑ & because power = force applied time the velocity at which the force is
applied, there must be an optimal speed of muscle shortening at which the most powerful
contraction is produced. After peak power is achieved, further ↑ in velocity result in a reduction
in power a muscle can generate.
 Production of maximum power is dependant upon the production of optimum force & optimum
velocity

312. What is Muscular Endurance?


 Ability of a muscle to exert tension over a period of time. Tension may be constant/varied;
longer the tension – greater the endurance. Training requires large number of reps against light
resistance.

313. List 5 common muscle injuries sustained during sporting pursuits.


 Strains: overstretching of muscle tissue
 Contusions: compressive force sustained during impacts causing muscle bruises
 Cramps: electrolyte imbalance, deficiencies in calcium/magnesium & dehydration cause severe
muscle spasm
 Delayed-Onset Muscle Soreness: arises 24-72 hours after participation in long/strenuous bout
of exercise ; microtearing of muscle tissue
 Compartment Syndrome: haemorrhage/oedema within muscle compartment resulting from
injury/excessive muscular exertion; pressure ↑ within compartment→ severe damage to
neuronal/vascular structures.

Downloaded by masoud masoudi (masoudmasoudi9777@gmail.com)

You might also like